You are on page 1of 36

LEGALEDGE TEST SERIES

MOCK COMMON LAW ADMISSION TEST 2023-24


MOCK CLAT 37
TR ID.

(In Figures)

INSTRUCTIONS TO CANDIDATES

Duration of Test : 2 Hours (120 Minutes)

c o m Maximum Marks : 120


1. Separate carbonised Optical Mark Reader
(OMR) Response Sheet is supplied along with
this Questions Booklet and the carbon copy .
10. Use BLACK/BLUE BALL POINT PEN only for

rs
writing the roll No. and other details on OMR
response Sheet.
has to be detached and taken by the
candidates.

k e
11. Use BLACK/BLUE BALL POINT PEN for
shading the circles. Indicate only the most

n
2. In case of any discrepancy in the question appropriate answer by shading from the

a
booklet (QB), please request the invigilator for options provided. The answer circle should be

r
replacement of a fresh packet of QB with OMR. shaded completely without leaving any space.
Do not use the previous OMR response Sheet 12. As the responses cannot be modified/corrected
for a fresh booklet so obtained.

o p
tr-5S7T5U3O8O
3. Candidates will not be given a second blank
1P5O on the OMR Response Sheet, candidates have
to take necessary precautions before marking

T
OMR response Sheet under any circumstance. the appropriate circle.
Hence, OMR response Sheet shall be handled 13. The candidate should retain the Admit Card
carefully. duly Signed by the invigilator, as the same has
4. Answer all questions. No clarification can be to be produced at the time of Admission.
sought on the Questions Paper 14. Handle the OMR response Sheet with care. Do
5. Possession of electronic devices in any form is not fold.
strictly prohibited in the examination Hall. 15. Ensure that invigilator puts his/her signature in
6. The use of any unfair means by any candidate the space provided on the OMR response
shall result in the cancellation of his/her Sheet. Candidate should sign in the space
examination. provided on the OMR response Sheet.
7. Impersonation is an offense and the candidate, 16. The candidate should write Question Paper
apart from disqualification, will be liable to be booklet No., and OMR response Sheet No.,
prosecuted. and sign in the space/column provided in the
8. The test Paper for Five Year integrated Law attendance sheet.
1E5J
5K7E5D3F8B
Programme is for 120 markstr-containing 120 17. Return the Original Page of OMR response
multiple Choice Questions. Sheet to the invigilator after the examination.
9. There will be Negative marking for multiple 18. The candidate shall not write anything on the
choice objective type questions. 0.25 marks OMR response Sheet other than the details
will be deducted for every wrong answer or required and in the spaces provided for.
where candidates have marked more than one
1P5O
tr-5S7T5U3O8O
response.
CONTENT OF QUESTION PAPER

Subject Q. No. Page No.


English Language 01-24 3
Current Affairs and General Knowledge 25-53 10
Legal Reasoning 54-83 15
Logical Reasoning 84-108 27
Quantitative Techniques 109-120 34

c o m
rs .
k e
r an
o p
tr-5S7T5U3O8O
1P5O

1E5J
tr-5K7E5D3F8B

1P5O
tr-5S7T5U3O8O

Head Office: 127, Zone II, MP Nagar, Bhopal |+91-7676564400| https://www.toprankers.com Page 2 of 36
SECTION-A : ENGLISH LANGUAGE

Directions (Q.1-Q.24): Read the passages carefully and answer the questions.

Passage (Q.1-Q.5): Recent social welfare regulations in India have tried to lean into this third more situated
point of view on the gig-economy, by offering a ‘middle path’ that treats gig-workers as independent
entrepreneurs with limited social security. The Code on Social Security specifies that the Central and State
governments shall frame welfare schemes for these workers in relation to life and disability cover, maternity
benefits, old age protection, provident fund, housing, etc. Given the complexity of fitting emerging kinds of work
within standard forms of employment, the lack of details is remiss. Maternity benefits, for instance, usually
depends on the fulfilment of a minimum length of service, and because gig-work tends to be flexible this can be
difficult to determine.

m
The other three codes on Wages; Industrial Relations; and Occupational Health, Safety and Working Conditions,

o
however, do not mention the new categories of gig or platform-worker. This potentially means that these workers

c
may be excluded from the right to unionize, minimum wages and assurances of safe-working conditions. Their

rs .
explicit inclusion is necessary because gig-work is often organized in a way that is not captured by what is
traditionally understood as an employer-employee relationship under labour law. For example, they are not
usually bound to one gig-based platform, which means that ascertaining who the employer is may be tricky.

k e
The benefits of regularized payment or even higher wages (compared to non-gig work) is short-lived given the
unpredictability of how much the gig-worker will be paid. Fair Work Foundation notes how platform companies

with or intimation to platform-workers.

r an
are opaque about work-allocation and ratings, and would often tweak payment structures without consultation

The COVID-19 pandemic has spotlighted

o
5S 7T 5U3O8O1P5O

pthe insecure conditions in which gig-workers operate and the lack of


a safety net provided by the law. A survey by the Indian Federation of App based Transport Workers (IFAT)
tr-
and The International Transport Workers Federation in Asia Pacific (IFT), of app-based drivers across India,

1.
T
reported that income of app-based drivers fell sharply during the devastating nation-wide lockdown in 2020.
Further, 95.3% of the respondents of the survey reported that they did not have any form of social security
including accidental, health or medical insurance. Alternatively, they seek to declare the relationship between
the aggregator and the platform-driver as an working relationship.

Which of the following is not a correct inference from the passage?


(a) The pandemic has shown that gig-workers work in conditions which are not very secure and outside legal
coverage.
(b) App-based drivers are also within the ambit of gig-workers and have admitted to not being covered by many
benefits.
(c) The codes on wages, industrial relations and occupational health has provisions for gig-workers.
(d) Giving regular payments or higher salary to workers results in only short-term gains for them.

2. Which of the following best describes author’s reasons for making an argument for explicit inclusion of gig-
1E5J
workers in law? tr-5K7E5D3F8B
(a) Because they are not considered permanent employees of the employers and therefore are outside of labour
laws.
(b) Because there is absence of employer-employee relationship between them and the employers; this keeps
them outside labour laws.
(c) Because there is no rule to determine the wages and other aspects of gig-workers as employees.
(d)5SBecause there
7T5U3O8O1P
5O is employee-employer relationship between them regardless of their status in labour law.
tr-

Head Office: 127, Zone II, MP Nagar, Bhopal |+91-7676564400| https://www.toprankers.com Page 3 of 36
3. What does the word “remiss” as used in the passage mean?
(a) Thoughtful. (b) Careful. (c) Negligent. (d) Diligent.

4. Based on the information set out in the passage, which of the following is most accurate?
(a) Only thirty percent drivers who were surveyed stated to have been covered by social security benefits.
(b) There is difficulty in inclusion of gig-workers in maternity benefits because they do not work for a minimum
continuous duration.
(c) Most labour codes mention the category of gig-workers in their ambit but there is no mention of employer-
employee relationship.
(d) Higher wages to gig-workers has led to positive developments and hope for long term gains.

5. Which of the following words will replace the bold word from the passage?
(a) Vicarious. (b) Fiduciary. (c) Occupational. (d) Employee- engager.

c o m
Passage (Q.6-Q.10): When I woke up, fecund morning light was slanting through every crack in the slat walls,

rs .
and it was already hot. My father was gone. So was his gun from the wall. I shook my brother awake and went
out to the cobblestone road without a shirt. The road and the stone steps were awash in the morning light.
Children squinting and blinking in the glare were standing vacantly or picking fleas out of the dogs or running

k e
around and shouting, but there were no adults. My brother and I ran over to the blacksmith’s shed in the shade
of the lush nettle tree. In the darkness inside, the charcoal fire on the dirt floor spit no tongues of red flame, the
bellows did not hiss, the blacksmith lifted no red-hot steel with his lean, sun-blackened arms. Morning and the

n
blacksmith not in his shop – we had never known this to happen. Arm in arm, my brother and I walked back

a
along the cobblestone road in silence. The village was empty of adults. The women were probably waiting at the

r
back of their dark houses. Only the children were drowning in the flood of sunlight. My chest tightened with
anxiety. Harelip spotted us from where he

o
5S 7T

p
5U3O8O1P5O
was sprawled at the stone steps that descended to the village fountain
and came running over, arms waving. He was working hard at being important, spraying fine white bubbles of
tr-
sticky saliva from the split in his lip.

“Heard?” I said vaguely.


T
“Hey! Have you heard?” he shouted, slamming me on the shoulder.
“Have you?”

“That plane yesterday crashed in the hills last night. And they’re looking for the enemy soldiers that were in it,
the adults have all gone hunting in the hills with their guns!”
“Will they shoot the enemy soldiers?” my brother asked shrilly.
“They won’t shoot, they don’t have much ammunition,” Harelip explained obligingly, “They aim to catch them!”
“What do you think happened to the plane?” I said.
“It got stuck in the fir trees and came apart,” Harelip said quickly, his eyes flashing. “The mailman saw it, you
know those trees.”
I did, fir blossoms like grass tassles would be in bloom in those woods now. And at the end of summer, fir cones
shaped like wild bird eggs would replace the tassles, and we would collect them to use as weapons. At dusk then
and at dawn, with a sudden rude clatter, the dark brown bullets would be fired into the walls of the storehouse. .
..
“Do you know the woods I mean?” tr-5K7E5D3F8B
1E5J

“Sure I do. Want to go?”


Harelip smiled slyly, countless wrinkles forming around his eyes, and peered at me in silence. I was annoyed.
“Nobody’s going! Kids are forbidden to go into the hills. You’d be mistaken for the alien soldiers and shot!”

1P5O
tr-5S7T5U3O8O

Head Office: 127, Zone II, MP Nagar, Bhopal |+91-7676564400| https://www.toprankers.com Page 4 of 36
6. What is the author’s main point in the passage above?
(a It is making a point of enemy soldiers being brought to justice in the place where the child resides.
(b) It is trying to depict the conversation between woman and curious children about a plane crashing with
enemy soldiers.
(c) It is stating the life of soldiers in the event of their being discovered in a foreign soil through the child’s
viewpoint.
(d) It is trying to depict the innocence and curiosity of children through their viewpoint in the background of
some armed conflict or war like situation.

7. What does the word “fecund” as used in the passage mean?


(a) To be Fertile. (b) To be Fruitful.
(c) Producing in Scarcity. (d) Bountiful.

8. Which of the following words will replace the bold word given in the passage?
(a) Indigenous. (b) Native. (c) Foreign. (d) Aboriginal.

c o m
9. Which of the following best describes the writing style of the passage?
(a) Reflective. (b) Descriptive. (c) Informative.

rs
(d) Analytical.
.
10. Which of the following is not a correct inference from the above passage?

k e
(a) There were some women staying in the houses while other adults had left for enemy search or other duties.

soldiers.

r n
(b) There was a plane crash in the hills due to some trees which had made the adults to go in search of enemy

a
(c) The narrator had begun suffering some form of anxiety while he was roaming around with his brother.
(d) The adults who had gone to search for

o
5O

p
the enemy soldiers would have most likely shot the soldiers.
tr-5S7T5U3O8O
1P

Passage (Q.11-Q.15): Absolute pacifism, then, is the idea that war is never justified. But does pacifism ever

T
work? Most often when people argue yes, they do so on the basis of one historical case study: Gandhi and Indian
Independence. The story usually goes that Mohandas Gandhi’s non-violent civil disobedience, notably in the
Salt March of 1930 and Quit India Movement in 1942, ultimately led to India’s independence in 1947. Without
Gandhi’s peaceful resistance, there would have been no independence. But, there are three problems with this
account.

First, Britain at the time (1947) was hardly ready to put up a determined fight for India. Not only had the British
Empire taken a military and economic battering from World War II, but decolonization was increasingly popular
in Britain itself (where the leftist Labour party, under Clement Attlee, was in charge).
Second, as Bertrand Russell points out, “When the enemy is resolute and brutal [absolute pacifism] has no
success. The Church persecuted heretics and Jews relentlessly, even when they made no attempt at armed
resistance. The Japanese, if they conquered India, would make short work of any movement of non-cooperation
on the part of Gandhi’s followers.” Non-violence, alone, does not win change so long as those in power have
guns and tanks and are not hesitant to use them.
1E5J
tr-5K7E5D3F8B
Finally, Gandhi was not the only actor involved in Indian independence. It is a bit of an oversight to ignore the
thousands of pro-Japanese, anti-British Indian Army troops who returned to India after World War II. While
Gandhi was no doubt important, you cannot help but feel that British colonial generals might have been more
concerned by India’s nationalistic army scattered around the country. The British philosopher, Bertrand Russell,
was not religious (quite the opposite), but he was a pacifist. Russell’s pacifism — what he called “relative
pacifism” —1Pis5Omore nuanced and perhaps more palpable. Russell argued two points:-
5S7T5U3O8O
tr-

Head Office: 127, Zone II, MP Nagar, Bhopal |+91-7676564400| https://www.toprankers.com Page 5 of 36
First, “that very few wars are worth fighting, and that the evils of war are almost always greater than they seem
to excited populations at the moment when war breaks out.” Even when the (purported) end goal is liberty, or
regime change, or justice, the death and misery inflicted by a war will, most often, outweigh the good. (Russell
was in favor of World War II, while being much maligned for his opposition to World War I.)

Second, there is a more general principle that “a civilized and humane way of life can hardly survive where wars
are frequent and serious.” This is perhaps the more idealistic notion that the world will never be better — a utopia
never reached — so long as we accept that wars can be good. Russell’s position is not that all wars are evil, but
that wars should be seen as evil far more often than they are.

Relative pacifism argues that some wars are justified and some fighting is morally right. Russell, for instance,
cites World War II and the French, Russian, and American revolutions as being such examples. If the end goal
is sufficiently righteous, and the destination can only be reached with war, then war it must be. Throughout

m
history, pacifists have been preaching their message of absolute non-violence, hoping to single-handedly end all

o
wars and bring about a utopian world without a hint of conflict. On the flip side, history has not been kind to
pacifism. Most of the time, it simply means you die at the hands of your enemies.

c
11. What is the central theme of the passage concerning pacifism and war ideologies?

rs .
(a) The historical efficacy of non-violent movements like Gandhi's in achieving independence.
(b) Bertrand Russell's advocacy for a utopian society free of all armed conflicts.

k e
(c) The potential benefits of certain wars in bringing about positive societal changes.
(d) The complexities and criticisms surrounding absolute pacifism as a practical approach to conflict

12.

an
What did Bertrand Russell's "relative pacifism" emphasize, as opposed to absolute pacifism?

r
(a) The need for a utopian society free of all armed conflicts.
(b) The importance of supporting all wars

o
5S 7T 5U3O8O1P5O

pto achieve societal changes.


(c) The recognition of some justifiable wars while cautioning against their overall negative impact.
tr-
(d) The belief that war is never a viable option for resolving conflicts.

13.
T
How does the historical example of Gandhi's non-violent civil disobedience and its impact on India's
independence strengthen the argument in favor of relative pacifism over absolute pacifism?
(a) The passage suggests that Gandhi's non-violent resistance would have failed against a resolute and brutal
enemy like the Japanese, highlighting the limitations of absolute pacifism in such scenarios.
(b) The passage highlights that India's independence was achieved primarily due to the efforts of pro-Japanese,
anti-British Indian Army troops, reinforcing the notion that peaceful resistance alone cannot bring about
significant change.
(c) Bertrand Russell's stance on relative pacifism indicates that only wars with highly righteous goals, such as
the French, Russian, and American revolutions, should be considered justifiable, aligning with the historical
significance of India's independence movement.
(d) The passage argues that the British Empire's weakened state after World War II was the sole factor leading
to India's independence, implying that absolute pacifism would have been just as effective in this context.

14. In the statement, "Throughouttr-history, 8B1E5J


5K7E5D3Fpacifists have been preaching their message of absolute non-violence,
hoping to single-handedly end all wars and bring about a utopian world without a hint of conflict," identify the
figure of speech?
(a) Metaphor. (b) Synecdoche. (c) Hyperbole. (d) Personification.

15. The passage highlights that absolute pacifism's proponents often face an uphill battle when advocating for their
cause. Which1P5O
idiom best captures this idea?
tr-5S7T5U3O8O
(a) Walking on thin ice. (b) Barking up the wrong tree.
(c) Swimming against the tide. (d) Cry over spilled milk.

Head Office: 127, Zone II, MP Nagar, Bhopal |+91-7676564400| https://www.toprankers.com Page 6 of 36
Passage (Q.16-Q.20): It’s astonishing how after years of birding, even aam-janta birds can keep you guessing:
These days when I set out for a walk around 5.30 am, the garden and lawns are resounding with birdsong – a
sort of city-bird morning chorus, if you may. Carefully, I try and single out the performers: bulbuls have their
individual mellifluous notes; there are a couple of magpie-robins playing their flutes at either end of the big lawn
and, thankfully, the jungle babblers haven’t woken up just yet. Occasionally, you hear the wheezy, husky chortles
of the yellow-footed green pigeons hiding out in the Satparni trees.
But these days what has caught my attention – and made me frown — is a fairly loud, continuous song that I
suspect belong to those lovely little, goggle-eyed yellow and ash-grey birds – the white-eyes. In a particular
corner of the large lawn, near a large mango tree, they drown the songs of all the others; and which brings us to
the issue: Generally white-eyes are soft-spoken birds that communicate with an often barely-audible background
jingling call or a sharpish ‘cheer’.

m
They are chiefly arboreal, hunting for insects, spiders, berries, and nectar among the blooms, often hanging

o
upside down on branches in order to conduct a thorough inspection. Outside of the breeding season – which may

c
stretch from February to September — they are sociable little birds (smaller than sparrows) gathering together
in flocks of even 50 or more.

rs .
They really do resemble very well brought-up little birds who take life rather seriously. (The white eye-ring

soft-voiced.

k e
makes them look as though they are wearing monocles in each eye and are somewhat scholarly!) And they are

n
The birds I hear in the lawn are anything but. They drown out even the magpie-robin’s fluting whistles. So was

a
this a rather bohemian little white-eye trying to outsing even the maestro? But again, it sounded like a chorus of

r
white-eyes that were singing, not an individual. Well, perhaps, if they were all madly in love and courting the

p
same girl, maybe each gentleman white-eye had enough testosterone coursing through his little body to make
him and all of them soundtr-like 5U3O
5S7Tan 8O1P5O
entire chorus.

o
That could account for the decibel level and the non-stop stream of songs – though now how would a lady tell

T
one dude from another and make her choice?! Maybe she’d just pick the sly one who kept shut and hopped up
next to her whispering sweet-nothings in her ear! Really, there is so much we have to discover regarding the love
life of even the commonest species!
16. Which of the following best represents the main idea of the passage?
(a) The author’s observations of various bird species and their distinct songs in the morning.
(b) The unusual loudness of white-eye birds’ songs and their dominance over other birds in a particular corner
of the large lawn.
(c) The habits and behaviors of white-eye birds, including their arboreal lifestyle and diet.
(d) The author’s musings on the love life and mate selection of different bird species.

17. “They are chiefly arboreal..” What does the word ‘arboreal’ refer to in context of the passage?
(a) Eating habits. (b) Habitat preference. (c) Hunting style. (d) Hours of activity.
8B1E 5J
18. Which of the following best describes the
tr-5K7E5D3F author’s attitude towards the loud songs of the white-eye birds?
(a) Irritation and annoyance. (b) Appreciation and admiration.
(c) Disapproval and criticism. (d) Curiosity and intrigue.

19. Which of the following is not true in the context of the passage?
(a) White-eyes are known for their soft-spoken communication.
(b) White-eyes
tr-5S7T 5U3O8O1P5Oprimarily feed on insects, spiders, berries, and nectar.
(c) White-eyes are smaller in size than sparrows.
(d) White-eyes are exclusively found in urban gardens and lawns.

Head Office: 127, Zone II, MP Nagar, Bhopal |+91-7676564400| https://www.toprankers.com Page 7 of 36
20. “But these days what has caught my attention – and made me frown — is a fairly loud, continuous song that I
suspect belong to those lovely little, goggle-eyed yellow and ash-grey birds – the white-eyes.” The sentence has
a grammatical error, find that out and choose the incorrect part among the following options.
(a) But these days what has caught my attention.
(b) and made me frown — is a fairly loud, continuous song.
(c) that I suspect belong to those lovely little, goggle-eyed yellow.
(d) and ash-grey birds – the white-eyes.

Passage (Q.21-Q.24): Sri Lanka’s strategic location in the Indian Ocean has long attracted attention from
regional rivals India and China. For years, free-flowing loans and infrastructure investments from Beijing helped
it gain the upper hand against New Delhi in the quest for influence. But the economic collapse allowed New
Delhi to swing the pendulum back in its favor, especially as China delayed its support for debt restructuring,
experts say. China owns about 10 percent of Sri Lanka’s foreign debt.

c o m
In a nod to the rising ties, Modi and Wickremesinghe hailed trade opportunities in energy and infrastructure.

Trincomalee, a northeastern coastal city in Sri Lanka, into an industrial hub.

rs .
That included the potential for a petroleum pipeline from southern India to Sri Lanka and developing

e
“We are seeing an increasing mode of competition, sometimes even conflict, between India and China playing
out in countries like Sri Lanka, where they’re often competing for the same projects on infrastructure, energy,

k
and even political influence in Sri Lanka,” Xavier said. It’s a power tug-of-war with both countries flexing and

all go pear-shaped for the island.

r an
strengthening their muscle as a currency to trade and spend, where it is no skin of their own and, in fact, could

Lanka’s ethnic minority Tamil 5U3O8O1P


tr-5S7Tpopulation

o p
The two leaders have expressed support5Oto fully implement an India-backed scheme to share power with Sri
in the island’s north and east provinces. The minority holds linguistic
and cultural ties with Tamils in southern India.

T
A civil war between the majority Sinhala-controlled Sri Lankan government and ethnic Tamil rebels killed at
least 100,000 people, according to U.N. conservative calculations, before it ended with the rebels’ defeat in 2009.
“We hope that the government of Sri Lanka will fulfill the aspirations of the Tamils,” Modi said.
Wickremesinghe said he presented Modi with a proposal for reconciliation and power sharing, saying he has
urged his parliament to reach a consensus and resolve the long-running conflict. His delegation also included
two Sri Lankan Tamil ministers.

Sri Lankan governments over the years have pledged to India that they will share more power with Tamils to
ensure peace and fully implement the 13th Amendment, which created provincial councils with a degree of
decentralized powers. But they have so far failed to do so, much to the dismay of both Tamil political leaders in
Sri Lanka and India.

“India must put pressure on Wickremesinghe and the opposition to be genuine and solve the problem,” said
Jehan Perera, a political analysttr-5K
based
7E5D3F 1E5J
in8BColombo.
The economy has shown signs of improvement since Wickremesinghe took over as president last year. Shortages
have been alleviated, power cuts have ended, and the rupee has begun to strengthen. But he has struggled to gain
the support of the opposition parties, which he needs to make progress on any power-sharing agreement.

“So much money can come from the Tamil diaspora if we fix this problem – a lot of diaspora Tamils are willing
to 5S
help if Sri Lanka
7T5U3O8O1P
5O treats its Tamil people fairly,” Perera added.
tr-

Head Office: 127, Zone II, MP Nagar, Bhopal |+91-7676564400| https://www.toprankers.com Page 8 of 36
Reference: Sri Lankan President’s Visit to India Signals Growing Economic and Energy Ties | Agreements on
connectivity, renewable energy and technology were signed during Ranil Wickremesinghe’s visit. By Krutika
Pathi and Bharatha Mallawarachi, The Diplomat, July 21, 2023]

21. Which of the following statements is true according to the author's perspective and take on the India-Sri Lanka
ties?
(a) The economic collapse in Sri Lanka was solely responsible for New Delhi gaining an upper hand against
Beijing in the quest for influence in the region.
(b) The civil war in Sri Lanka between the Sinhala-controlled government and ethnic Tamil rebels ended with
the defeat of the government forces in 2009.
(c) The rising ties between India and Sri Lanka have primarily focused on the development of infrastructure and
energy projects, while political influence remains a secondary consideration.
(d) Despite Sri Lanka's promises to India about power-sharing with the Tamil population, there has been a

leaders in both countries.

c o m
consistent failure to fully implement the 13th Amendment, leading to discontent among Tamil political

22.

rs .
Identify the gist of the author's take on the solution to the Sri Lankan economic crisis.
(a) The author suggests that the Sri Lankan government should seek more financial aid from India and China to
overcome the economic crisis.

Tamil population as a potential solution to the economic crisis.

k e
(b) The author emphasizes the importance of implementing the India-backed scheme for power-sharing with the

(c) The author believes that Sri Lanka should focus on strengthening its trade ties with regional rivals India and
China to revive its economy.

an
(d) The author highlights the need for Sri Lanka to attract investments from the Chinese diaspora to address the
economic crisis.

r
23. In which of the given sentences
tr-5S7T5U

o
3O
has
p 8O1P5O
the underlined word been used in the same manner as in the given line – "A
civil war between the majority Sinhala-controlled Sri Lankan government and ethnic Tamil rebels killed at least

T
100,000 people, according to U.N. conservative calculations, before it ended with the rebels’ defeat in 2009."?
(a) The glooming forecast are based on overly conservative projections of growth
(b) The politician adopted a conservative approach to garner support from traditional voters
(c) The natives of Tamil Nadu and Kerala tend to be socially conservative.
(d) The artist's conservative brushstrokes reflected a minimalist yet profound style.

24. Which of the given options correctly identifies the figure of speech used along with the meaning of the said in
the lines –
"It’s a power tug-of-war with both countries flexing and strengthening their muscle as a currency to trade and
spend, where it is no skin of their own and, in fact, could all go pear-shaped for the island."
(a) Personification - It attributes human-like qualities to the countries, portraying them as flexing and
strengthening their muscle (economic power) in a currency-like manner for trade and spending, which can
impact the island.
(b) Hyperbole - It exaggerates the intensity of the competition between the countries, portraying it as a tug-of-
war with immense economic tr-5Kimplications,
7E5D3F8B1E5J which may lead to disastrous outcomes for the island.
(c) Simile - It implies that both countries are engaging in a fierce competition like a tug-of-war to exert influence
over the island, using their economic power as a currency for their ambitions.
(d) Metaphor - It compares the competition between the countries to a tug-of-war, indicating a fierce struggle
for power and influence, and their strengthening economic power to flexing muscles, symbolizing the display
and utilization of their economic strength.
1P5O
tr-5S7T5U3O8O

Head Office: 127, Zone II, MP Nagar, Bhopal |+91-7676564400| https://www.toprankers.com Page 9 of 36
SECTION -B : CURRENT AFFAIRS, INCLUDING GENERAL KNOWLEDGE

Directions (Q.25-Q.53): Read the information carefully and answer the questions.

Passage (Q.25-Q.29): Black Sea Grain Initiative


As world leaders gather in Delhi on Friday ahead of the G-20 Summit, United Nations Secretary-General
Antonio Guterres is on a mission to revive the Black Sea Grain Initiative (BSGI or BSI), in talks on the sidelines
of the summit. The deal that facilitates export of grain from Russia and Ukraine lapsed in July. Among those in
Delhi, Turkey President Recep Erdogan, Russian Foreign Minister Sergey Lavrov, as well as leaders of the
European Union and UN officials will be trying to work out a compromise that will give assurances to Russia,
for an exchange that will allow grain export from blockaded Ukrainian ports to restart at the earliest, sources
told The Hindu.
Ministry of External Affairs (MEA) officials said they were unaware of any meetings on the sidelines of the G-

25.
20 Summit, but underlined India’s position in support of the initiative.

What was the primary purpose of the Black Sea grain deal?

c o m
(a) To ensure safe passage for grain ships from European Union
(b) To restrict Russia’s oil exports
(c) To ensure safe passage for grain ships from Ukraine

rs .
26.
(d) To ensure safe passage for grain ships from Crimea

How many Ukrainian ports were specifically mentioned in the Black Sea grain deal?

k e
27.
(a) Two (b) Four

r an
(c) Three

Why has Russia decided not to renew the Black Sea grain deal?
(d) Five

o
5S 7T 5U3O8O1P5O

p
(a) Russia believes food prices have stabilized globally
(b) Russia felt it was unnecessary as Ukraine had enough resources
tr-
(c) due unkept promises and barriers in exporting its agricultural products

28.
T
(d) Ukraine was not exporting enough grain

What was the length and width of the safe passage in the Black Sea under the grain deal?
(a) 310 nautical miles long and 5 nautical miles wide
(b) 310 nautical miles long and 3 nautical miles wide
(c) 300 nautical miles long and 3 nautical miles wide
(d) 320 nautical miles long and 4 nautical miles wide

29. EU is considering connecting a subsidiary of which Russian bank to SWIFT for grain and fertiliser transactions?
(a) Sberbank (b) Gazprombank (c) VTB Bank (d) Rosselkhozbank

Passage (Q.30-Q.34): Amendment to the Information Technology Rules, 2021


While the 2021 rules covered social media intermediaries, the 2023 amendments substituted the terms “social
media intermediary and significant social media intermediary” with “a social media intermediary, a significant
social media intermediary and tr-an5Konline gaming
7E5D3F8B 1E5J intermediary”.
There is also some overlap between online games and social media intermediaries — like games available or
accessed through social media platforms or using social media accounts that allow posting or sharing of
achievements etc.
The new rules define an online gaming intermediary to mean any intermediary that "enables the users of its
computer resource to access one or more online games". The additional portions provide the regulatory
framework for online gaming and misinformation.
1P5O
tr-5S7T5U3O8O
online cnbc

Head Office: 127, Zone II, MP Nagar, Bhopal |+91-7676564400| https://www.toprankers.com Page 10 of 36
30. What does the term "online gaming intermediary" refer to as per the 2023 amendments?
(a) Any intermediary offering video streaming
(b) Any intermediary that offers only casino games
(c) Any intermediary that enables users to access one or more online games
(d) Any intermediary that allows betting on online games

31. What type of online game, specifically mentioned in the 2023 amendments, is still allowed?
(a) Betting on game outcomes (b) Real money online game
(c) Non-permissible online game (d) Online games involving wagering

32. Who will verify if an intermediary is hosting non-permissible online real money games or misleading
advertisements?
(a) The individual user (b) Social media platforms

33.
(c) MeitY's fact-checking body (d) Game developers

c o m
Intermediaries are required to take action within what timeframe upon receiving complaints about specific
content?
(a) 12 hours (b) 24 hours (c) 48 hours (d) 72 hours

rs .
34.
(a) Rajeev Chandrasekhar
(c) Piyush Goyal
(b) Nitin Gadkari
(d) Nirmala Sitharaman

k e
Who holds the position of Union Minister of State for Electronics and Information Technology?

Passage (Q.35-Q.39): National Innovation Survey

r an
The detailed analysis of the survey results also provides valuable insights on the innovation ecosystem in India.
Thus, I anticipate this report to be of great
innovation and economic development,” he said.

o
tr-5S 7T

p
5U3O8O1P5O
interest to policymakers, researchers, and practitioners in the field of

T
He further highlighted that these learnings can add significant value to the Make-in-India programme objective,
specifically the Production Linked Incentive (PLI) schemes, positioned to boost manufacturing in a variety of
sectors, including electronics, pharmaceuticals, and automobiles, that are already yielding results.
“The NMIS study & findings will contribute to strengthening the baselines to some of capacities and capabilities,
opportunities and challenges in the manufacturing value-chains that require immediate attention,” Dr.
Chandrasekhar pointed out.

It is a joint study by the Department of Science and Technology (DST) and the [x] to evaluate the innovation
performance of manufacturing firms in the India.
It was conducted as a [y] survey that examined the innovation processes, outcomes, and barriers in manufacturing
firms and also studied the innovation ecosystem that affects innovation outcomes in these firms.
This study is a follow-up of DST’s first National Innovation Survey held in [1].

35. In the National Manufacturing Innovation Survey, which state has been ranked as the most 'innovative' state?
(a) Maharashtra (b) Tamil 5D3F8B1E5J (c) Gujarat
Nadu
tr-5K7E (d) Karnataka

36. The “National Manufacturing Innovation Survey (NMIS) 2021-22: Summary for Policymakers” aimed to
evaluate the innovation performance of manufacturing firms in India. This survey was conducted jointly by the
Department of Science and Technology (DST) and __________.
(a) World Trade Organization (WTO)
(b)5S7T
International
5U3O8O1P5O
Monetary Fund (IMF)
tr-
(c) World Bank
(d) United Nations Industrial Development Organization (UNIDO)

Head Office: 127, Zone II, MP Nagar, Bhopal |+91-7676564400| https://www.toprankers.com Page 11 of 36
37. How many distinct components were included in the NMIS 2021-22 survey?
(a) One component (b) Two components (c) Three components (d) Four components

38. What was India's rank in the Global Innovation Index among 132 countries in 2022?
(a) 20 (b) 40 (c) 60 (d) 80

39. When was the first National Innovation Survey conducted?


(a) 2005 (b) 2008 (c) 2011 (d) 2014

Passage (Q.40-Q.44): Article 370


Jammu and Kashmir was accorded special status because of the circumstances in which it acceded to India soon
after Independence. Sandwiched between the two new Dominions of India and Pakistan, the State did not make

m
an immediate decision on which country to join, as it had vital economic and cultural links with both. Maharaja

o
Hari Singh, in August 1947, offered to have ‘Standstill Agreements’, which meant that existing arrangements

c
will continue on all matters, with both Pakistan and India. Pakistan agreed immediately. India offered to negotiate
a ‘Standstill Agreement’, but no such pact materialised.

rs .
However, by October 1947, the Maharaja was faced with regular military attacks from the Pakistan side by

k e
“soldiers in plainclothes, desperadoes with modern weapons”. There was mass infiltration by heavily armed
tribesmen from the North-West Frontier into Kashmir, and the Maharaja felt this could not happen without the
support of the governments of Pakistan and of the North-West Frontier Province. In a desperate letter to India’s

n
Governor-General, Lord Mountbatten, on October 26, 1947, he sought help from India, noting that “naturally

a
they cannot send the help asked for” without his State acceding to the Dominion of India. Therefore, he attached
an ‘Instrument of Accession’ with the letter.

r
40. What was the objective behind

o
5U3O
tr-5S7Tthe

p 8O1P5O
inclusion of Article 370 in the Indian constitution?
(a) To grant complete independence to Jammu and Kashmir.

41.
(d) None of these
T
(b) To integrate Jammu and Kashmir fully with the Indian Union.
(c) To provide special autonomy and privileges to Jammu and Kashmir.

On October 17, 1949, Article 370 was introduced into the draft constitution by __________ as Article
__________.
(a) Alladi Krishnaswami Ayyar; Article 307 (b) D.P. Khaitan; Article 306 A
(c) Mohammad Saadulla; Article 307 (d) N Gopalaswami Ayyangar; Article 306 A

42. In which of the following year the Constitution (Application to Jammu and Kashmir) Order issued by the
President, which led to the withdrawal of the special status of Jammu and Kashmir and the extension of all
provisions of the Indian Constitution to the region?
(a) 2016 (b) 2017 (c) 2018 (d) 2019

43. Which of the following articletr-of5Kthe


7E5DIndian Constitution empowers the Jammu & Kashmir legislature to define
3F8B1E5J
the permanent residents of the state, and their special rights and privileges?
(a) Article 30A (b) Article 35A (c) Article 36A (d) Article 37A

44. How many Lok Sabha seats will be allotted to Jammu and Kashmir and Ladakh after the division of the state
into two Union Territories?
(a) 4 seats for Jammu and Kashmir, 2 seats for Ladakh.
3O8O1P5O
(b) 5 5U
tr-5S7T seats for Jammu and Kashmir, 1 seat for Ladakh.
(c) 3 seats for Jammu and Kashmir, 3 seats for Ladakh.
(d) 3 seats for Jammu and Kashmir, 3 seats for Ladakh.
Head Office: 127, Zone II, MP Nagar, Bhopal |+91-7676564400| https://www.toprankers.com Page 12 of 36
Passage (Q.45-Q.49): Myanmar Issues
The 10-member Association of Southeast Asian Nations (ASEAN) has finally shown the courage to call out the
junta in Myanmar for the ongoing violence and its failure in implementing the Five-Point Consensus that was
reached between the two sides, aimed at addressing the post-coup crisis in the country. Myanmar has seen a
security and economic decline ever since the military ousted the democratically elected government of Aung
San Suu Kyi in February 2021. Under the ASEAN plan, reached in April 2021, the junta had promised to stop
the violence and start inclusive political dialogue, but has continued violating the consensus. ASEAN kept
pushing the generals, but stopped short of antagonising them. However, the latest statement from ASEAN, issued
after its annual summit in Jakarta, suggests a hardening of its stand. It has “strongly condemned” the continued
acts of violence, and directly urged the “armed forces in particular and all related parties concerned... to ... stop
targeted attacks on civilians”.
The Hindu

45. When did the military overthrow the government of Aung San Suu Kyi in Myanmar?
(a) April 2021 (b) February 2021 (c) August 2021 (d) January 2021

c o m
46. What was the intention of the Five-Point Consensus between ASEAN and the junta?
(a) To restore Aung San Suu Kyi to power

rs .
(b) To stop the violence and start inclusive political dialogue
(c) To provide Défense assistance
(d) To elevate Myanmar's chairmanship in ASEAN

k e
47.
(a) Malaysia (b) Indonesia

r an
Which country will take over the ASEAN chairmanship that Myanmar was to assume in 2026?
(c) Thailand (d) The Philippines

48.

o
7T5U3O
tr-5Sproposed
What is the primary solution
(a) Military's abrogation
p 8O1P5O
for Myanmar's crisis?

49.
(a) 1980
T
(b) Immediate UN peacekeeping intervention
(c) The formation of a new alliance for democracy
(d) Restoration of a legitimate, responsible, and responsive regime

In which year did Burma officially change its name to Myanmar?


(b) 1989 (c) 1995 (d) 2000

Passage (Q.50-Q.53): Special Protection Group


Special Protection Group (SPG) Director and Kerala-cadre IPS officer Arun Kumar Sinha, 61, who died in
Gurgaon on Wednesday after a prolonged battle with cancer, had heralded modernisation in the Kerala Police
and launched several initiatives during his stint with the BSF.

Sinha was made SPG Director in 2016. On May 30, this year, a day before his retirement, the term of the 1987-
batch officer was extended for another year. Kerala Chief Minister Pinarayi Vijayan expressed condolences on
Sinha’s demise, saying he wastr-an efficient
5K7E
5J
5D3F8B1Eofficer who had executed many responsibilities in the state police.
Sinha, who hailed from Hazaribagh in Jharkhand, began his career as assistant superintendent of police at
Mananthavady in Kerala’s Wayanad in 1989. Later, he was posted as SP in various sensitive districts and also
served as the Commissioner of Police in Thiruvananthapuram and Kochi. He had been IG for South Zone and
the Police Administration at the headquarters. Sinha had been credited with successfully heading various crime
investigation teams and solved many cases.
The Hindu 1P5O
5S7T5U3O8O
tr-

Head Office: 127, Zone II, MP Nagar, Bhopal |+91-7676564400| https://www.toprankers.com Page 13 of 36
50. Which significant system did Sinha introduce in the Kerala Police?
(a) Cyber Crime Cell (b) Crime Stopper System (1090)
(c) Police Helpline 100 (d) Women's Safety Cell

51. Which initiative under Sinha's leadership helped restrict the intrusion of Pak nationals in the Harami Nala area?
(a) Establishment of the coastal police unit
(b) Implementation of the Crime Stopper System
(c) Creation of the Amphibious Commando Team known as Creek Crocodile
(d) Introduction of specialized training apparatus in the SPG

52. What did Sinha do to strengthen the professional acumen of SPG officers?
(a) Established the country's first-ever police website
(b) Introduced the Crime Stopper System
(c) Introduced numerous specialized training apparatus/programs
(d) Formed the first Marine Battalion of BSF

c o m
53.

rs .
Which of the following committee recommended setting up a special protection unit which was later re-
christened to SPG?
(a) Janaki Ram Committee
(c) Kelkar Committee
(b) N.N. Vohra Committee
(d) Birbal Nath Committee

k e
r an
o p
tr-5S7T5U3O8O
1P5O

1E5J
tr-5K7E5D3F8B

1P5O
tr-5S7T5U3O8O

Head Office: 127, Zone II, MP Nagar, Bhopal |+91-7676564400| https://www.toprankers.com Page 14 of 36
SECTION – C: LEGAL REASONING

Directions (Q.54-Q.84): Read the comprehension carefully and answer the questions.

Passage (Q.54-Q.58): Section 74 of the Indian Contract Act, 1872, principally lays down the law governing
liquidated damages. Essentially, it states that parties at the time of contracting may stipulate an amount in the
agreement itself that shall become payable on breach of contract by the defaulting party in favour of the innocent
party. In cases where the amount stipulated in the contract is likely to be a genuine pre-estimate of damages
flowing from the breach, it is called 'liquidated damages', otherwise the amount is labelled 'penalty'. It is also
often believed that 'penalty' is the sum stipulated in the contract to deter the breach. The idea behind incorporating
this provision in the main statute was to avoid litigation and add commercial certainty. It seeks to offer threefold
protection: 1) For the injured party, it may afford the only possibility for compensation of loss that is not

m
susceptible to proof with sufficient certainty; 2) For the party in breach, it has the effect of limiting damages to

o
the sum stipulated; 3) For society as a whole, it seeks to save the time of judges, witnesses, and parties and
thereby reduces the cost of litigation.

c
relief-act-201203

rs .
Source: https://www.livelaw.in/columns/indian-contract-act-section-74-liquidated-damages-penalty-specific-

54.

k e
Choose a statement that is not untrue with regards to the information provided in the passage:
(a) Loss is sine qua non for the award of damages under this head, and proof of actual loss is mandatory in all
cases except when it is difficult or impossible to prove that the stipulated amount is a genuine pre-estimate
of damages.

an
(b) In all cases, reasonable compensation will be fixed based on the well-known principles of mitigation and
causation specified in Section 73.

r
compensation cannot be

o
tr-5S 7T5U3O8O1P
awarded.
p
(c) The amount stipulated in the contract5Oas liquidated damages is just an upper ceiling, or the bar beyond which

(d) While Section 74 fulfils its purpose for the defendant by limiting the damages to the sum stipulated, for the

55. T
innocent party it falls short of remedying the commercial uncertainty, leaving the determination of the
compensable amount largely within the realm of the court or tribunal.
Ecoworld engaged Dobler for the design, supply, and installation of façade and glazing works in the building,
which contained Blocks A, B, and C. The contract itself did not distinguish between sections, and the contract
did not contain any provision for sectional completion of the works. The date for completion was changed from
21 August 2017 to 30 April, 2018 by a variation deed. Ecoworld took partial possession of Blocks B and C on
15 June, 2018 with practical completion of the remainder of the works being certified on 20 December, 2018.
Ecoworld levied liquidated damages from 30 April, 2018 to 20 December, 2018. Decide.
(a) Dobler is liable to pay the sum stipulated by Ecoworld for breach of contract.
(b) Dobler is not liable to pay any damages to Ecoworld for breach of contract.
(c) Dobler is not liable to pay Ecoworld liquidated damages as Ecoworld took partial possession of blocks B
and C with practical completion of the remainder of the works being certified on 20 December, 2018.
(d) Dobler is not liable to pay any damages as the Contract did not contain any provision for sectional completion
of the works. tr-5K7E5D3F8B
1E5J

56. The plaintiff agreed to supply potatoes to the respondent. The plaintiff deposited earnest money to demonstrate
his seriousness about contracting with the respondent. Later, plaintiff made a default in supplying potatoes, and
so the contract was rescinded by respondent. The deposit of earnest money has been forfeited. Choose a correct
statement in light of the information presented in the passage:
(a) No loss was suffered by the respondent due to the non-performance of the contract by the plaintiff.
(b)
tr-5SThe 8O1P5O
respondent
7T5U3O is justified in rescinding the contract but the amount cannot be forfeited.
(c) The provisions of Section 74 are applicable.
(d) The plaintiff is guilty for breach of contract.
Head Office: 127, Zone II, MP Nagar, Bhopal |+91-7676564400| https://www.toprankers.com Page 15 of 36
57. A contract for a university dorm rental state: “Students who cancel their dormitory housing agreement after
moving into their room shall pay damages amounting to $5.00 per day for the remainder of the rental term”. Is
the clause valid?
(a) Yes, as the university has all rights to speculate on the possibility of students defaulting on their dormitory
agreement.
(b) Yes, as the $5.00 per day charge is considered reasonable, according to market value calculations.
(c) No, since the damages are difficult to quantify.
(d) No, because the stipulated sum is a penalty rather than liquidated damages.

58. What, according to your interpretation of the passage, is the closest possible meaning of liquidated damages?
(a) A pre-estimate of the amount of damages to be awarded in the event of a breach of contract occurring.
(b) Damages that are quantified by the courts following a breach.

excessively high in order to encourage performance and deter breaches.


(d) Damages that are negotiated and agreed upon by the parties following a breach.

c m
(c) A pre-estimated amount that a party will need to pay should he breach the contract. The amount is set

o
rs .
Passage (Q.59-Q.63): Section 319 to 338 of the Indian Penal Code deal with hurt and grievous hurt in various
forms. Section 319 of the Indian Penal Code, 1860 (hereinafter “IPC”), defines hurt as: “Whoever reasons bodily
pain, disorder, or disease to any man or woman is said to have caused harm.”
To constitute hurt, any of the following essentials need to be caused:
 Bodily pain, or

k e
 Disease, or
 Infirmity to another.

r an
Whereas, Section 320: The following kinds of hurt only are designated as “grievous”, Emasculation, Permanent
privation of the sight of either eye,

o
tr-5S7T5U3O8O
joint, destruction or permanent
p
Permanent
1P5O privation of hearing in either ear Privation of any member or
impairing of the powers of any member or joint, Permanent disfiguration of the
head or face, Fracture or dislocation of a bone or tooth, Any hurt that endangers life or renders the sufferer

T
incapable of carrying out his usual activities for a period of twenty days. Section 322 of the IPC characterizes
‘deliberately causing grievous hurt’ as follows: Whoever deliberately causes hurt, if the hurt which he expects
to cause or realises that he will generally be prone to cause is grievous hurt, and if the hurt which he causes is
grievous hurt, is said to have: “wilfully to cause grievous hurt.”
Voluntarily causing hurt: Section 321, Whoever does any act with the intention of thereby causing hurt to any
person, or with the knowledge that he is likely thereby to cause hurt to any person, and does thereby cause hurt
to any person, is said to have: “voluntarily to cause hurt”.
(Source: https://blog.ipleaders.in/hurt-and-grievous-hurt-everything-you-need-to-know-about-
it/#:~:text=Section%20319%20of%20the%20Indian,the%20offence%20of%20inflicting%20harm.)

59. Which of the following illustrations clearly demonstrates Section 321 of the IPC?
I. Deepak threw corrosive stones Hasina for declining his employment bid.
II. The accused caused damage to the neck of the victim from behind.
III. Deepak intentionally sets out to shock Nitin with a weak coronary heart and succeeds in doing so.
3F8B1E5J
7E5Dshe
IV. The accused was a prostitute,
tr-5Kand inflicted syphilis to her customers.
(a) I, II, and III (b) II and III (c) III (d) I, III, and IV

1P5O
tr-5S7T5U3O8O

Head Office: 127, Zone II, MP Nagar, Bhopal |+91-7676564400| https://www.toprankers.com Page 16 of 36
60. Among the wounds inflicted on the complainant by the accuser’s clenched fists and lathi were a break in one
side of his thumb caused by his fall to the ground during the accused's assault. The court prescribes punishment
for causing grievous hurt. Choose a correct statement.
(a) The punishment is appropriate since the accused's actions resulted in "grievous hurt".
(b) The punishment is appropriate because the complainant's thumb was broken on one side.
(c) The punishment is improper because the complainant's thumb was broken as a result of his own fall on the
ground.
(d) The punishment is erroneous since the defendant had no intention of causing grievous hurt to the
complainant.

61. Dherya was in charge of the Janki hospital's mental asylum unit. Mayank, one of the patients, pulled her hair and
hurled her to the ground in a moment of insanity. With his hand, he also struck her on the head. She has filed an

has endured. Decide whether the accused will be held liable for hurt or grievous hurt?

c m
FIR against the accused, requesting that the individual receive the harshest sentence possible for the trauma she

o
(a) The intention of the accused was to cause grievous hurt to the complainant, and hence he is liable for the
same.

rs .
(b) The accused is not liable for grievous hurt because Mayank’s action caused bodily injury to Dherya.
(c) The accused is liable for hurt, as his intention was to cause hurt.

62.

k e
(d) The accused is liable for hurt, as he caused bodily pain and mental trauma to the accused.

Bhavna and Neelam went to the Ministry of Crabs for dinner. A live kitchen is also available at the popular

n
restaurant, where chefs demonstrate their master culinary skills in front of the guests. Playing with 4-5 knives at

a
once was one such act. One of the knives struck Bhavna in the ears when the chefs were performing, causing

r
excessive blood loss. To stop the bleeding, the doctors had to cut the ear and perform surgery. What crime will
the chef be charged with?
tr-5S7T5U

o
(a) The chef will be held liable
8O1P5O

p
for3Othe offence of hurt, and not grievous hurt.
(b) The chef will be held liable for grievous hurt.

63.
T
(c) The chef will not be held liable as the guests consented to the risk involved in the said act.
(d) The chef will not be held liable for the act, as he had no intention to cause either Hurt or grievous hurt.

Jethalal was annoyed by his tenant's misbehavior, Bhide. To scare the tenant's wife, Jethalal made a piercing
sound and stretched his arms towards her, brandishing a gun, with the intention of forcing the couple to leave
the premises. She passed out from shock when the bullet struck her. She became unwell for a long period of
time. Determine whether Jethalal is liable in this instance. The act was sufficient to cause a state of temporary
mental impairment resulting in disorder of the mind to attract hurt as defined in section 319. Justify.
(a) No, Jethalal should be held liable under Section 321 rather 319 as there was intention on part of Jethalal to
cause hurt to the couple.
(b) Yes, as Jethalal caused mental disorder to the wife of Bhide hence liable under section 319.
(c) Yes, as Jethalal should be held liable under Section 319 rather 321 of Indian penal Code.
(d) No, as Jethalal had no intention to cause such mental infirmity. Hence, his conviction under Section 319 is
justified.
1E5J
tr-5K7E5D3F8B

1P5O
tr-5S7T5U3O8O

Head Office: 127, Zone II, MP Nagar, Bhopal |+91-7676564400| https://www.toprankers.com Page 17 of 36
Passage (Q.64-Q.69): The concept of ‘natural law’ is one of the most fundamental and influential ideas in
jurisprudence. It refers to the idea that there is a higher law or moral order that transcends human-made laws and
norms and that governs the conduct and affairs of human beings. Natural law theorists believe that natural law
is derived from reason, nature, or divine revelation and that it provides the basis for human rights, justice, and
morality. They also believe that natural law is universal, objective, and binding on all human beings, regardless
of their time, place, or culture. When we study the concept of natural law, we will encounter many principles
that have been developed and debated by various thinkers and schools of thought throughout history. Now, let
us look into some of the basic principles of natural law that have been proposed by different natural law theorists.
• The principle of rationality: This principle states that natural law is based on human reason, which can
discover the universal and immutable truths about human nature and morality. This principle was advocated
by classical thinkers like Plato, Aristotle, and Cicero, who believed that reason was a gift from God or a
reflection of the divine logos. It was also supported by modern thinkers like Thomas Aquinas, John Locke,

o m
and Immanuel Kant, who argued that reason was a natural faculty that enabled human beings to discern their
natural rights and duties, and to act according to the categorical imperative or the law of nature.

c

rs .
The principle of naturalism: This principle states that natural law is based on the observation of nature, which
reveals the order and harmony of creation and the natural rights and duties of human beings. This principle
was supported by ancient Stoics, who viewed nature as a rational and providential cosmos that endowed

k e
human beings with a natural law that was in accordance with their nature. It was also endorsed by medieval
scholastics, who integrated Aristotelian philosophy with Christian theology, and by Enlightenment
philosophers like Hugo Grotius, Samuel Pufendorf, and Jean-Jacques Rousseau, who asserted that natural

r n
law was a social contract that derived from the state of nature or the natural condition of human beings.

a
The principle of theonomy: This principle states that natural law is based on the revelation of God, who is
the source and judge of all laws and

o
tr-5S 7T

p
5U3O8O1P5O
morals. This principle was espoused by biblical scholars, Christian
theologians, and Islamic jurists, who claimed that natural law was a reflection or manifestation of God’s will
or commandments. They maintained that natural law was revealed through scriptures, prophets, or

64.
T
messengers, and that it was superior to human laws or customs. They also held that natural law was a guide
or criterion for evaluating the validity or morality of human laws or actions.
[An Overview of the Natural School of Law, Lakshay Kumar, https://blog.ipleaders.in/natural-school-of-law ,
accessed on July 7, 2023.]

During a criminal trial, Kelly, a judge, sentences a defendant guilty of murder to death. . She intentionally applies
the law of the land, that prescribes capital punishment for such an offence. Kelly’s intention behind this act is to
uphold justice and deterrence and to respect the sovereignty of the state. In the given situation, does Kelly’s
action potentially conform to the principle of theonomy in natural law theory?
(a) No, Kelly’s action does not conform to the principle of theonomy in natural law theory, as she does not act
according to the revelation of God, who is the source and judge of all laws and morals.
(b) Yes, Kelly’s action potentially conforms to the principle of theonomy in natural law theory as she acts
according to the revelation of God, who is the source and judge of all laws and morals.
(c) No, Kelly’s action does not conform to the principle of theonomy in natural law theory, as she acts according
to human reason or nature,tr-which 1E5Jsufficient sources of natural law.
3F8Bnot
5K7E5Dare
(d) Yes, Kelly’s action potentially conforms to the principle of theonomy in natural law theory as she acts
according to human reason or nature, which are reflections or manifestations of God’s will or
commandments.

1P5O
tr-5S7T5U3O8O

Head Office: 127, Zone II, MP Nagar, Bhopal |+91-7676564400| https://www.toprankers.com Page 18 of 36
65. Jack, a scientist, during a research project, discovers a new vaccine that can cure a deadly disease. He
intentionally publishes his findings and shares his data with the scientific community and the public. Jack’s
intention behind this act is to contribute to the advancement of knowledge and to benefit humanity. In the given
situation, does Jack’s action potentially conform to the principle of rationality in natural law theory?
(a) No, Jack’s action does not conform to the principle of rationality in natural law theory, as he does not act
according to human reason, which can discover the universal and immutable truths about human nature and
morality.
(b) Yes, Jack’s action potentially conforms to the principle of rationality in natural law theory, as he acts
according to human reason, which can discover the universal and immutable truths about human nature and
morality.
(c) No, Jack’s action does not conform to the principle of rationality in natural law theory, as he acts against his
duty as a scientist and violates the intellectual property rights of his research institution.

m
(d) Yes, Jack’s action potentially conforms to the principle of rationality in natural law theory, as he acts

o
according to the categorical imperative or the law of nature that requires him to treat others as ends and not
as means.

c
66.

rs .
Isla, a social worker, during a humanitarian crisis, provides food and water to refugees who are fleeing from war
and persecution in their home country. She intentionally shares her resources and skills with those who are in

e
need and vulnerable, regardless of their nationality, religion, or ethnicity. She also helps them to find shelter,
medical care, and legal assistance. Isla’s intention behind this act is to show compassion and solidarity and to

k
uphold human dignity and rights. In the given situation, does Isla’s action potentially conform to the principle
of naturalism in natural law theory?

an
(a) No, Isla’s action does not conform to the principle of naturalism in natural law theory, as she does not act

r
according to the observation of nature, which reveals the order and harmony of creation and the natural rights
and duties of human beings.
tr-5S7T5U3O
(b) Yes, Isla’s action potentially

o
8O1P5O

p
conforms to the principle of naturalism in natural law theory as she acts
according to the observation of nature, which reveals the order and harmony of creation and the natural rights

T
and duties of human beings.
(c) No, Isla’s action does not conform to the principle of naturalism in natural law theory, as she acts according
to human reason or revelation, which are not sufficient sources of natural law.
(d) Yes, Isla’s action potentially conforms to the principle of naturalism in natural law theory, as she acts
according to human reason or revelation, which are reflections or manifestations of nature.

67. Harry, a hacker, during a cyber attack, exposes the confidential data of a multinational corporation that is
involved in illegal and unethical activities. He intentionally leaks the data to the public and the media, revealing
the corruption and fraud of the corporation. Harry’s intention behind this act is to expose the truth and to demand
justice and accountability. In the given situation, does Harry’s action potentially conform to the principle of
theonomy in natural law theory?
(a) Yes, Harry’s action does not conform to the principle of theonomy in natural law theory as he does not act
according to the revelation of God, who is the source and judge of all laws and morals.
(b) Yes, Harry’s action potentially conforms to the principle of theonomy in natural law theory, as he acts
according to the revelationtr-of
5KGod, 8B1E5J
7E5D3Fwho is the source and judge of all laws and morals.
(c) No, Harry’s action does not conform to the principle of theonomy in natural law theory.
(d) Yes, Harry’s action potentially conforms to the principle of theonomy in natural law theory, as he acts
according to human reason or nature, which are reflections or manifestations of God’s will or
commandments.

1P5O
tr-5S7T5U3O8O

Head Office: 127, Zone II, MP Nagar, Bhopal |+91-7676564400| https://www.toprankers.com Page 19 of 36
68. Emma, a journalist, during a civil war, exposes the atrocities committed by the government forces against the
rebels and civilians. She intentionally publishes a series of articles and photographs that document human rights
violations and war crimes. Emma’s intention behind this act is to inform the public and the international
community about the truth and to demand justice and accountability. In the given situation, does Emma’s action
potentially conform to the principle of theonomy in natural law theory?
(a) No, Emma’s action does not conform to the principle of theonomy in natural law theory, as she does not act
according to the revelation of God, who is the source and judge of all laws and morals.
(b) Yes, Emma’s action potentially conforms to the principle of theonomy in natural law theory, as she acts
according to the revelation of God, who is the source and judge of all laws and morals.
(c) No, Emma’s action does not conform to the principle of theonomy in natural law theory, as she acts according
to human reason or nature, which are not sufficient sources of natural law.
(d) Yes, Emma’s action potentially conforms to the principle of theonomy in natural law theory, as she acts

commandments.

c o m
according to human reason or nature, which are reflections or manifestations of God’s will or

69.

rs .
Carol, a doctor, during a medical emergency, performs a life-saving surgery on a patient without obtaining his
consent. She intentionally acts in the best interest of the patient, who is unconscious and has no relatives or
friends present. Carol’s intention behind this act is to save the patient’s life and prevent any harm or injury. In

e
the given situation, does Carol’s action potentially conform to the principle of rationality in natural law theory?
(a) Yes, Carol’s action potentially conforms to the principle of rationality in natural law theory, as she acts

k
according to the universal and immutable truth that human life is valuable and sacred.

n
(b) No, Carol’s action does not conform to the principle of rationality in natural law theory, as she violates the

a
patient’s autonomy and right to consent or refuse medical treatment.

r
(c) Yes, Carol’s action potentially conforms to the principle of rationality in natural law theory.
(d) No, Carol’s action does not conform5O

o
5S 7T 5U3O8O1P

p
to the principle of rationality in natural law theory, as she acts according
to her own subjective judgment and not according to reason.
tr-

T
Passage (Q.70-Q.74): The Supreme Court will hear on August 25, the State of Tamil Nadu's application seeking
fresh directions for the release of Cauvery River water from Karnataka. A bench of Justices BR Gavai, PS
Narasimha and Prashant Kumar Mishra will hear the application.

The plea for urgent directions was mentioned before Chief Justice of India DY Chandrachud led the bench on
Monday. The CJI had then assured that he would list the matter soon. Tamil Nadu has sought fresh directions on
the release of water from Karnataka, claiming that the neighbouring state has "changed its stand" and has released
only a reduced quantum of 8,000 cusecs of water as against 15,000 cusecs that was agreed to earlier. The
application by the Tamil Nadu government was filed days after Chief Minister Stalin wrote to Prime Minister
Narendra Modi, seeking his intervention in the matter.

Article 262(1) provides that Parliament may adopt legislation for the settlement of disputes or complaints
concerning the use, distribution, or control of transboundary waters in a river or river valley. According to Article
262(2), Parliament may adopt a law which may impede the jurisdiction of the Supreme Court or of any other
court in relation to the dispute/appeal 3F8B1E5J to in Article 262 (1).
tr-5K7E5Dreferred
[Source: https://www.barandbench.com/news/litigation/supreme-court-hear-tamil-nadus-plea-cauvery-river-
water-dispute-august-25]

1P5O
tr-5S7T5U3O8O

Head Office: 127, Zone II, MP Nagar, Bhopal |+91-7676564400| https://www.toprankers.com Page 20 of 36
70. The Cauvery River originates in Karnataka and flows through Tamil Nadu before entering the Bay of Bengal.
The dispute revolves around the allocation of water from the river to meet the agricultural, industrial, and
domestic needs of both states. The conflict over the Cauvery River waters had been a source of tension and legal
battles for decades. Both Karnataka and Tamil Nadu had been seeking a fair and equitable share of the river
water for their respective regions. The dispute was characterized by sporadic violence, protests, and tensions
between the two states' governments and their populations. Who can the states approach to resolve the dispute?
(a) They can approach the Supreme Court or any authority appointed by Parliament.
(b) They can approach the High Court and then the Inter- State Tribunal.
(c) They can approach the Inter-State River Tribunal constituted under Article 262.
(d) They can approach the Supreme Court compulsorily under Article 32.

71. Haryana, being a downstream state of the Sutlej River, relies on its waters for various purposes, including

m
irrigation, agriculture, and domestic consumption. Punjab, an upstream state, also utilises the Sutlej's waters for

o
its agricultural and industrial needs. However, a disparity in water availability during different seasons and years,

c
coupled with increasing demands in both states, led to conflicts over the sharing of the river's waters. The court

of the prescribed amount. What can the other state do to enforce the full order?
(a) They can file a writ petition in the High Court.

rs .
orders 7,000 cusecs of water to be released. They release 4000 cusecs of water and refuse to release the whole

(b) They can file an application with the Supreme Court.


(c) They can file an appeal in the Supreme Court under Article 136.
(d) They can file a reversionary appeal in the Supreme Court.

k e
72.

an
The Bhakra Dam was built primarily to address irrigation and power generation needs in the northern regions of

r
India. The dam's construction was a joint effort by the governments of Punjab (now Punjab and Haryana) and
Rajasthan, with Himachal Pradesh hosting

o
5S 7T 5U3O8O1P5O

pthe dam itself. The Sutlej River originates in Tibet and flows through
Himachal Pradesh before entering Punjab and Haryana. During the monsoon and post-monsoon periods, the
tr-
Sutlej River receives substantial inflows, leading to the need for regulated water release to prevent flooding

T
downstream. The dam's operators, in collaboration with the concerned state governments and the central
government, manage the water release from the Bhakra Dam. In the case of a water dispute, what can be the
remedy in light of the passage?
(a) There can be action in the Supreme Court or any other authority constituted for the purpose.
(b) There can be action in the River Dispute Tribunal under Article 262.
(c) There can be action as per the State Acts responsible for the same.
(d) There can be action in the High Court under Article 226.

73. The Sutlej-Yamuna Link (SYL) Canal project aimed to provide water to Haryana from the Sutlej River, which
flows through Punjab. The project was initially proposed to address the water scarcity in Haryana, which was
created after the reorganization of states in 1966. However, the canal's construction faced various delays and
obstacles due to disputes over water-sharing agreements between Punjab and Haryana. With the dispute
escalating and affecting the agricultural and economic activities of both states, the central government convenes
meetings and discussions between the leaders and officials of Punjab and Haryana. Mediation is offered to
5J
7E5D3F8B1Ebut
encourage both states to reach tr-a 5K
consensus, it is not successful. What will be the next course of action?
(a) There can be a case in the authority created by the Parliament.
(b) There can be a case in the High Court for river water settlement.
(c) There can be a case in the Supreme Court under Article 136 of the Constitution.
(d) There can be a case in the Supreme Court under Article 133 of the Constitution.

1P5O
tr-5S7T5U3O8O

Head Office: 127, Zone II, MP Nagar, Bhopal |+91-7676564400| https://www.toprankers.com Page 21 of 36
74. The Godavari River, one of the major rivers in India, flows through multiple states, including Maharashtra and
Andhra Pradesh. The construction of various dams and reservoirs along the river has led to disputes over the
sharing of water between these states. The Polavaram Dam, situated in Andhra Pradesh, is one such significant
dam that has caused controversies due to its potential impact on water availability downstream in Maharashtra.
If there are problems related to water sharing, what will be the remedy in light of the passage?
(a) There can be a case in the Supreme Court for enforcement of the civil water dispute under Article 262.
(b) There can be a case in the Supreme Court for the prevention of the release of aqua creatures.
(c) There can be a case in the Supreme Court for the matters of the natural resources.
(d) There can be a case in the Supreme Court for the enforcement of river disputes under Article 262.

Passage(Q.75-Q.79): Article 25(1) gives all persons the freedom to practice, profess, and propagate one's
religion subject to public order, morality, health, and other provisions of the Part, and 25(2)(a) empowers the

m
State to regulate or restrict those activities of any religious practice that are economic, political, financial in

o
nature, or any other activity that is secular, and 25(2) (b) allows for the formulation of social welfare, reform,

c
and opening up of religious places of public type for all sections of Hindus by legislation. It can only be claimed

rs .
against the government and statutory bodies. This freedom of practice involves adhering to all kinds of rituals
and practices that go along with belief and religion. When the individual makes the declaration of his conscience,
it takes the countenance of professing his belief, and when this declaration becomes an invitation to others to

k e
join this belief by persuasion and awareness, it takes the shape of propagation. But sometimes this thin line is
crossed, and persuasion takes the form of compulsion or deception, which is prohibited. The Essential
Practice Test is devised by the Supreme Court to determine which religious practices are essential or fundamental

n
to a particular religion and which are just superstitions. In the Shirur Mutt case, the Supreme Court held that "a

a
religion may not only lay down a code of ethical rules for its followers to accept; but it might also prescribe

r
rituals and observances, ceremonies, and modes of worship which are regarded as integral parts of religion, and
these forms and observances might extend

o
5S 7T

p
5U3O8O1P5O
even to matters of food and dress". The Supreme Court held in
the Shayara Bano case that triple talaq is not an essential practice of Islam and proclaimed it unconstitutional.
tr-
The concept of secularism is not similar to the doctrine of secularism in America, which tries to establish a rigid

75.
T
distinction between the state and religion. India adopts an affirmative side of secularism, wherein it is neutral in
terms of religion and is in consonance with its ancient legacy of accommodating all faiths and abandoning none.
[Source: https://articles.manupatra.com/article-details/Freedom-of-Religion-under-Indian-Constitution]

There is a small village in India with a predominantly Hindu population. The villagers practice a unique form of
worship, which involves performing a traditional dance as a part of their religious rituals. The dance involves
using a particular type of drum, which is considered essential to the practice. One day, the Co-operative Union
in which the village is located announced a ban on the use of the drum in public places, citing noise pollution
concerns. The Hindu villagers argue that the use of the drum is an essential part of their religious practice and
that the ban violates their right to practice their religion as guaranteed by the Indian Constitution. Decide in light
of the passage:
(a) The use of drums is an essential practice of religion, and it violates their right to practice practicing religion.
(b) The Noise pollution is a valid concern under public order, and therefore, rights can be curtailed.
(c) The right can be claimed to be violated because religious rituals are considered essential practices as per the
1E5J
Shirur case. tr-5K7E5D3F8B
(d) The right cannot be claimed because it does not meet the criteria for freedom of religion.

1P5O
tr-5S7T5U3O8O

Head Office: 127, Zone II, MP Nagar, Bhopal |+91-7676564400| https://www.toprankers.com Page 22 of 36
76. There is a small city in India with a diverse population of Hindus, Muslims, Christians, and Sikhs. The local
government decides to organize a public celebration for a national festival, which involves unfurling the national
flag on important buildings and a big feast in the community hall. It coincides with a Hindu festival, and large
statues of Hindu gods are also erected in the nearby public square by people. The authorities state that the
celebration is open to all members of the community, regardless of their religion, and that it is not meant to
exclude or discriminate against any religious community. The Muslim and Christian communities in the city
express concern about the celebration, arguing that it is not inclusive of their religious beliefs and that it violates
the secular nature of the Indian Constitution. Decide in accordance with the passage:
(a) The celebration violates the secular nature of the Constitution because the state should remain neutral
towards religion.
(b) The celebration does not violate the secular nature of the Constitution because there is representation of all
communities.

promoted by the government.

c m
(c) The celebration violates the secular nature of the Constitution because a particular community is being

o
(d) The celebration does not violate the secular nature of the Constitution because it is impartial in nature.

77.

rs .
A group of religious missionaries from a particular religion arrive in a small village in India. They begin to
preach and offer incentives to the villagers to convert to their religion. They give the villagers clothes, a weekly

k e
ration, and cash in the thousands to win their approval and embrace their religion willingly. Some of the villagers
are won over by them, but it is not meeting their targeted numbers. They begin running schools with quality
English education at nominal cost. It attracts a lots of crowds from villagers. The schools run by these

n
missionaries begin awarding very low marks to students who belong to different religion. They run counselling

a
classes for those students and state that a belief in their religion has drastically increased the marks of many

r
students. Students in bulk convert while also urging their parents. Some people complain to authorities, and
decide in accordance with the passage: 5O
(a) They are not covered by

o
tr-5S 7T5U
the 3O8O1P
freedom
p
of religion because they were propagating using dishonorable means.
(b) They are covered by freedom of religion because they used persuasion as a method, which is permissible.

78.
T
(c) They are not covered by the freedom of religion because they used deceit in their methods, which is not
allowed.
(d) They are covered by freedom of religion because there is no element of compulsion in their methods.

Murugan Temple is situated at Murugan Hill in Pathanamthitta district, Merala, India. It is a Hindu temple
dedicated to Lord Murugan and his followers open to the public. This temple is managed and administered by
the statutory body, Mravancore Devaswom Board created under the Mravancore-Mochin Hindu Religious
Institution Act, 1950. Women belonging the ages of 10 and 50 years are not permitted to enter the temple. This
restriction on women is justified on the ground that Lord Murugan is a celibate and for the preservation of the
character of deity Murugan, it is necessary. A petition under Art 32 of the Indian Constitution was filed in the
Supreme Court by the registered association of Indian Young Lawyers seeking entry of women between 10 and
50 years old. Will the petition succeed, and why?
(a) The petition will not succeed because the Board is not a government authority against whom freedom of
religion can be claimed.
(b) The petition will succeed because the 1E5J
8Bfreedom of religion of women is being violated by the ban on entry.
tr-5K7E5D3F
(c) The petition will succeed because it includes the power to throw open religious Hindu public places for all
types of people.
(d) The petition will not succeed because the women are being restricted under public order.

1P5O
tr-5S7T5U3O8O

Head Office: 127, Zone II, MP Nagar, Bhopal |+91-7676564400| https://www.toprankers.com Page 23 of 36
79. Many members of the Hureshi Community belong to the Muslim religion and have lived in India for decades.
They mainly engage in slaughtering cattle for consumption by people of their religion, especially during festivals.
According to their religion, this is considered valid, and cattle slaughter is basically done for essential purposes
at a major festival. Another major religion, Hinduism, considers cows to be pious animals, believing that they
have the auspicious presence of God. Cows are protected for milk and other products. There is a disease that
breaks out among cows and can be transmitted to humans on consumption. One state government passed a law
to prohibit cow slaughter in all parts of the state. The Hureshi community challenges this law, stating that their
freedom of religion is being violated and that it is an essential part of their religion. Decide as per the above
passage:
(a) The Hureshi community will succeed because cow slaughter is an essential part of their religion.
(b) The law will be upheld because it protects the freedom of religion of the majority community.
(c) The law will not be upheld because it violates freedom of religion of the Hureshi community.
(d) The law will be upheld because it is covered by the restrictions to freedom of religion.

c o m
Passage(Q.80-Q.84): Malicious prosecution is the malicious intention of unsuccessful criminal or bankruptcy
or liquidation proceedings against another without reasonable or probable cause.

rs .
In the case of West Bengal State Electricity Board v. Dilip Kumar Ray, the Court defined the term “malicious
prosecution” in the following words:-

probable cause to sustain it is a malicious prosecution.”

k e
“A judicial proceeding instituted by one person against another, from wrongful or improper motive and without

Following are the essential elements which the plaintiff is required to prove in a suit for damages for malicious
prosecution:-
 Prosecution by the defendant.
 Absence of reasonable and probable cause.

r an
 Defendant acted maliciously.
 Termination of proceedings

o
tr-5S7T5U

p
8O1P5O
in3Othe favour of the plaintiff.
 Plaintiff suffered damage as a result of the prosecution.

T
The word “prosecution” carries a wider sense than a trial and includes criminal proceedings by way of appeal,
or revision but does not include discharge due to lack of adequate proof. The Court held that merely bringing the
matter before the executive authority did not amount to prosecution and, therefore, the action for malicious
prosecution could not be maintained. The question relating to want of reasonable and probable cause in a suit
for malicious prosecution should be decided on all facts before the Court. Malice need not just be a feeling of
enmity, spite or ill will or spirit of vengeance but it can be any improper purpose which motivates the prosecutor,
such as to gain a private collateral advantage. The damage can be classified under three categories namely: to
reputation or, person or property. There is generally compensation awarded in the action for malicious
prosecution.
[Source: https://blog.ipleaders.in/malicious-prosecution-law-tort/]

80. In the town of Riverton, there was a well-respected and influential businessman named Thomas Anderson.
Thomas was known for his successful chain of retail stores and his active involvement in local politics. The
situation began when a disgruntled former employee named Robert Smith, who had been fired from one of
7E5D3F8B1E5J
Thomas's stores, decided to seektr-5Krevenge. Robert harbored deep resentment towards Thomas, believing that he
had been unjustly dismissed. Consumed by anger and a desire for retribution, Robert hatched a plan to ruin
Thomas and bring him down. Robert went to the local police station and fabricated a story, alleging that Thomas
had been embezzling funds from his own company. He presented false financial documents, manipulated to
implicate Thomas in a complex fraud scheme. Additionally, Robert convinced a few other ex-employees, who
held grudges against Thomas for various reasons, to testify against him. The police, believing they had
substantial evidence,
1P5O swiftly arrested Thomas. He is treated with due regard to his health and well-being. News
tr-5S7T5U3O8O
of the arrest spread like wildfire throughout Riverton, causing shock and disbelief among the townspeople who

Head Office: 127, Zone II, MP Nagar, Bhopal |+91-7676564400| https://www.toprankers.com Page 24 of 36
had respected and admired him. The case verdict was in favour of Thomas. Later, he filed a case of malicious
prosecution against Robert. On what ground will he succeed in light of the passage?
(a) There was damage to his person through the malicious case without reasonable cause by Robert, whose
verdict came in his favour.
(b) There was damage to his reputation through the malicious trial without reasonable cause by Robert, whose
decision came in his favour.
(c) There was damage to his property through the malicious trial without reasonable cause by Robert, whose
decision came in his favour.
(d) There was damage to his profession through the malicious trial without reasonable cause by Robert, whose
decision came in his favour.

81. John, a well-respected and successful businessman, finds himself at the center of a controversial case. It all
begins when a rival businessman, Robert, becomes envious of John's accomplishments and reputation within the

m
industry. Determined to bring John down, Robert hatches a plan to tarnish his name. Robert bribes a former

o
employee, Lisa, who had previously worked for both John and Robert's companies. Lisa, feeling resentful after

c
being let go by John due to her poor performance, agrees to play along. She presents a story claiming that John

rs .
forced her to engage in unethical practices during her employment. Robert takes Lisa's allegations to the
investment regulatory authorities with executive powers of search and seizure and presents them as solid
evidence against John. He orchestrates a media campaign, ensuring that the accusations gain widespread

k e
attention, damaging John's personal and professional reputation. The news outlets portray John as a corrupt
businessman, creating a negative perception among his colleagues, clients, and the public. The authority does
not find John guilty. Thereafter, John brings a case of malicious prosecution against Robert. Will he succeed?
Decide in light of the passage:

an
(a) Yes, he will succeed because there was damage to his reputation with a malicious case without reasonable
cause and a decision in his favour.

r
(b) No, he will not succeed because8O
tr-5S7T5U3O

o
(c) Yes, he will succeed because
the

p
requirements of prosecution are not being met.
1P5O
all requirements for malicious prosecution are being met.
(d) No, he will not succeed because there was no damage in this case.

82.
T
John Davis, a respected businessman and philanthropist, found himself entangled in a web of accusations. One
sunny afternoon, John received an urgent phone call from his lawyer, Emma Thompson. She informed him that
he was being charged with embezzlement, a crime he vehemently denied. The charges were initiated by his
longtime business partner, Robert Barnes, with whom John had built a successful software development
company. Robert had been embezzling funds from their company for years, creating a paper trail that
conveniently pointed towards John's involvement. As the legal proceedings began, it became clear that the
evidence against John was circumstantial at best. However, the prosecution was relentless in their pursuit,
determined to make an example out of the prominent businessman. The court perused all the evidence and gave
the decision in favour of Robert. The decision is widely reported, tarnishing his reputation. Later, there was an
appeal in which John’s guilt was absolved and he was cleared of all charges. John wants to bring a case of
malicious prosecution against Robert. Will he succeed in light of the passage?
(a) No, he will not succeed because the decision by the court did not come in his favour.
(b) Yes, he will succeed because the appeal is in his favour which is included in the prosecution.
3F8B1E5J
tr-5K7E5D
(c) No, he will not succeed because there was no damage that happened as a result of the prosecution.
(d) Yes, he will succeed because one of the requirements of malicious prosecution are being met.

1P5O
tr-5S7T5U3O8O

Head Office: 127, Zone II, MP Nagar, Bhopal |+91-7676564400| https://www.toprankers.com Page 25 of 36
83. In a small town called Meadowbrook, there was a highly respected lawyer named Rebecca Thompson. She was
known for her integrity and dedication to justice. One day, a wealthy businessman named Richard Morgan filed
a complaint against Sarah Lewis, a hardworking single mother who ran a small bakery in town. Richard claimed
that Sarah's bakery had sold him a batch of contaminated pastries, causing him severe illness. He accused Sarah
of negligence and demanded a substantial amount of compensation. The local police, without conducting a
thorough investigation, arrested Sarah based solely on Richard's accusation. This is reported in local newspapers,
resulting in the picketing of Sarah’s bakery. Rebecca takes her case and fights it vehemently, resulting in a verdict
in Sarah’s favour revealing that Richard wanted to shut down her bakery for his prospective bakery to succeed.
Sarah files a case of malicious prosecution against Richard. What is the essential element of malicious
prosecution that was met by a flimsy investigation by the police?
(a) It was the presence of prosecution as per the criteria.
(b) It was the absence of reasonable cause on the part of Richard.
(c) It was the presence of damage to reputation to Sarah.
(d) It was a decision in favour of Sarah.

c o m
rs .
k e
r an
o p
tr-5S7T5U3O8O
1P5O

1E5J
tr-5K7E5D3F8B

1P5O
tr-5S7T5U3O8O

Head Office: 127, Zone II, MP Nagar, Bhopal |+91-7676564400| https://www.toprankers.com Page 26 of 36
SECTION D: LOGICAL REASONING

Directions (Q.84-Q.108): Read the passage carefully and answer the questions.

Passage (Q.84-Q.88): In the dimly lit psychology lab at the University of Cambridge in 1966, a young graduate
student named Nicholas Humphrey was engrossed in an experiment that would later shape his understanding of
Consciousness. He was studying the superior colliculus, a brain area involved in visual processing, in an
anesthetized monkey. The monkey's brain cells were active, suggesting that some form of vision might be
possible without conscious sensation – a feeling experienced by a person.

This led Humphrey to a monkey named Helen, whose visual cortex – a part of brain that processes information
from eyes – had been removed, leaving her superior colliculus intact. Over time, Helen learned to navigate her
surroundings and even demonstrated depth perception, despite her lack of conscious visual sensations. This

m
phenomenon, known as "blindsight," raised profound questions about the nature of consciousness. If

o
nonconscious perceptions were sufficient for survival, why did humans evolve to experience such rich and varied
sensations?

c
rs .
Humphrey's exploration of these questions led him to propose that our ability to have conscious experiences
shapes our objectives in ways that are evolutionarily advantageous. Sensations not only motivate us but also

k e
enable sensation-seeking activities like play, exploration, and imagination, which have helped us learn more
about ourselves and thrive. They also make us better social psychologists, allowing us to understand the feelings
and motives of others by consulting our own.

an
In his new book, "Sentience: The Invention of Consciousness," Humphrey argues that the more mysterious and

r
unworldly the qualities of phenomenal consciousness, the more significant the self becomes. This significance,
in turn, increases the value we place on our

o
5S 7T

p
5U3O8O1P5O
own and others' lives. He quotes the poet Byron, who wrote that "the
great object of life is Sensation—to feel that we exist—even though in pain," reflecting his own wide-ranging
tr-
life experiences, from studying gorillas with primatologist Dian Fossey to editing the literary journal Granta.

T
Humphrey's views on consciousness subtly challenge many current ideas. For instance, he believes that
artificially intelligent machines, which are all perception and no sensation, will never achieve sentience – the
feeling. Similarly, he argues that animals such as reptiles and insects, which face little evolutionary pressure to
develop a grasp of other minds, are also unlikely to be sentient. Understanding the practical value of sentience,
he suggests, will help us recognize that we use our sensations to better enjoy and understand ourselves, one
another, and the world.

84. The author is likely to agree with which one of the following?
(a) Our experiences encourage us to seek more experiences that help us evolve.
(b) Humans get impacted by non-conscious perceptions more than they do by conscious perceptions.
(c) Perceptions that are non-conscious impact our survival and are detrimental to our evolution.
(d) Consciousness motivates us to seek more non-conscious perceptions that help us evolve.

85. Which of the following is an assumption


tr-5K7E5D3F8Bon which Humphrey’s argument depends?
1E5J

(a) Conscious visual sensation of humans will be no different than that experienced by the monkeys.
(b) The results of the perception of the humans will be same as that of the anesthetized monkey.
(c) The results of the visual experiment on the anesthetized monkey will not be different than that on humans.
(d) The internal body structure of monkeys in the experiment above was the same as that of humans.

1P5O
tr-5S7T5U3O8O

Head Office: 127, Zone II, MP Nagar, Bhopal |+91-7676564400| https://www.toprankers.com Page 27 of 36
86. Humphrey’s statement that “artificially intelligent machines, which are all perception and no sensation, will
never achieve sentience” will get most strengthen if it were known that
(a) Artificially intelligent machines work the same way as humans
(b) Perception can’t be had without conscious sensations.
(c) Artificially intelligent machines are not the same as humans.
(d) Sentience can be achieved with conscious sensations.

87. Which of the following weakens Humphrey’s argument in the passage?


(a) Other famous scientists believe that sight has no role in evolution of humans.
(b) In a study, it was found that some people were able to navigate well despite lack of vision.
(c) In a survey of people who lost all their limbs, people reported that their life experiences were as rich and
varied as normal people.

conscious vision.

c o m
(d) In a study, it was reported that most people didn’t know the difference between non-conscious and

88. The tone of the passage can best be described as:


(a) Dismissive and critical.
(c) Sarcastic and humorous.
(b) Objective and informative.
(d) Pessimistic and gloomy.

rs .
k e
Passage (Q.89-Q.93): Coca-Cola is one of the world’s most widely recognized brands, operating in more than
200 countries. This global reach creates a significant carbon footprint. The company uses over 200,000 vehicles

an
for daily product distribution and runs hundreds of bottling plants and syrup factories worldwide. However,
Coke’s single largest contribution to climate change comes from its refrigeration equipment.

r
o
5S 7T 5U3O8O1P5O

p
Running refrigerators uses a lot of electricity, and some coolants in these systems are greenhouse gases that trap
heat in the atmosphere. Almost two-thirds of the climate impact of refrigeration comes from electricity
tr-
consumption, and refrigerants account for the rest. As of 2020, refrigeration produced nearly 8% of global
greenhouse gas emissions.

T
Coca-Cola’s marketing strategy emphasizes that a cold Coke should always be within reach. This notion is
deeply ingrained in the company, which is obsessed with making sure Coca-Cola is always within “an arm’s
reach of desire,” as one Coke president put it. Major companies like Coca-Cola have profited handsomely by
making their products readily available worldwide, creating a fast-paced, long-distance form of commerce that
is a major driver of our planet’s current ecological crisis.

Refrigerants first became an environmental issue because of concerns about ozone loss, not climate change.
Before the 1980s, the primary coolants used in refrigerators were chlorofluorocarbons, or CFCs. These
compounds were odorless, nonflammable, and seemingly nontoxic – all properties that made them useful to
industry. However, in the 1970s, researchers found that CFCs could destroy stratospheric ozone, a gas in the
atmosphere that protects life on Earth from the Sun’s ultraviolet radiation. This led to the ban of CFCs through
the 1987 Montreal Protocol, one of the most successful environmental treaties on record.
1E5J
tr-5K7E5D3F8B
Chemical companies such as DuPont led the way in promoting new chlorine-free refrigerants, called
hydrofluorocarbons or HFCs, that would not deplete the ozone layer. Like CFCs, HFCs appealed to industry
because they were odorless, nonflammable, and posed no serious threats to human health. However, HFCs had
a big drawback: They were powerful greenhouse gases that trapped heat in the Earth’s atmosphere, warming the
planet’s surface. Some HFCs had warming impacts more than 1,000 times greater than carbon dioxide, the most
abundant greenhouse
5S7T5U3O8O
1P5O gas.
tr-

Head Office: 127, Zone II, MP Nagar, Bhopal |+91-7676564400| https://www.toprankers.com Page 28 of 36
In an interview with Coca-Cola’s former chief sustainability officer, Jeff Seabright, I asked him whether the
company had ever considered thinking more broadly about the necessity of cooling all those Cokes around the
clock. Seabright’s response was an emphatic “No,” and that the company was still driven by the mantra of
making Coke available for immediate consumption at the point of sale.

Despite the resources that Coca-Cola has invested in changing refrigerants, its cooling equipment is still
warming our planet. As I see it, perhaps it’s time for Coke to question whether it needs all those machines in
the first place – and for consumers to consider whether their have-it-now expectations are worth the
environmental costs they impose.

89. Which of the following is NOT an assumption that has been made by the author in the passage?
(a) Coca-Cola’s business is not going to be impacted sufficiently so as to make this transition worthless.
(b) Buying cokes and refrigerating it later for consumption will have lesser impact on environment compared
to that of current practice.

o m
(c) The impact of company's global success and marketing strategy is not as big as that of the strategies of
other businesses.

c
on global greenhouse gas emissions.

rs .
(d) A significant reduction in the use of refrigeration equipment by Coca-Cola would have a noticeable impact

90.
undertaken by the company, would most significantly address the author's concerns?

k e
Given the author's critique of Coca-Cola's impact on environmental, which of the following changes, if

(a) Coca-Cola decides to invest in renewable energy sources to power its bottling plants and syrup factories.

n
(b) Coca-Cola introduces a new line of products that do not require refrigeration.

a
(c) Coca-Cola reduces the number of vehicles used for daily product distribution by optimizing its supply
chain.

r
91.
(d) Coca-Cola launches a global campaign

o
5O

p
to promote the recycling of its product packaging.
tr-5S7T5U3O8O
1P

Suppose a new study reveals that consumers are highly aware of the environmental costs of their consumption

T
habits and are willing to change these habits. How would this finding challenge the assumptions underlying the
author's argument?
(a) It would suggest that Coca-Cola's marketing strategy is not as influential as the author assumes.
(b) It would imply that the environmental impact of refrigeration is less significant than the author suggests.
(c) It would indicate that the author's suggestion for consumers to reflect on their consumption habits is
unnecessary.
(d) It would undermine the author's assertion that Coca-Cola's global operations contribute significantly to its
carbon footprint.

92. Which of the following situations represent neither an analogy nor a paradox of the author’s arguments?
(a) A fast-food chain promotes the convenience of drive-thru service, leading to an increase in vehicle idling
and air pollution.
(b) A clothing brand promotes fast fashion, leading to increased waste due to the rapid turnover of clothing
items.
(c) A city promotes the use oftr-public 3F8B1E5J
5K7E5Dtransportation but continues to build infrastructure that favors private car
use.
(d) A software company develops a new application that requires high processing power, leading to increased
energy consumption by devices running the app.

1P5O
tr-5S7T5U3O8O

Head Office: 127, Zone II, MP Nagar, Bhopal |+91-7676564400| https://www.toprankers.com Page 29 of 36
93. The author’s suggestion that it’s time for Coke to question whether it needs all those machines in the first place
– and for consumers to consider whether their have-it-now expectations are worth the environmental costs they
impose, would get most weakened if it were known that
(a) Some refrigerants are not as harmful as others in terms of damage to the environment and greenhouse
effect.
(b) technology allows building energy efficient cooling appliances that don’t use refrigerants.
(c) Refrigerants not only negatively impact environment but also humans in the form of severe respiratory
issues.
(d) In a study analysizing the impact of refrigerants on the environment, it is was not clear how much is the
impact of refrigerants on the environment.

Passage (Q.94-Q.98): As political theatre, it was hard to beat Rahul Gandhi’s drive from Washington to New

m
York last week in an enormous truck driven by an Indian immigrant. A movie of this journey has been posted

o
on social media by his publicity team, and I recommend it. Rahul comes across as charming, humble, sincere,

c
and genuinely interested in the driver’s account of life on the road and in the Punjabi songs he likes to listen to
on his long, lonely drives.

rs .
Spokesmen for the BJP and some of Modi’s ministers have been virulent and venomous in their attacks on

k e
Rahul’s speeches and travels in the United States. They have gone out of their way to find reasons to prove that
the heir to the mighty Nehru-Gandhi dynasty is obsessed with getting foreigners to intervene in Indian affairs.
When, in one of his speeches, he made fun of the Prime Minister for prostrating himself before the ‘sengol’, they

n
dug up a photograph of Rahul doing a full ‘sashtang pranam’ to portray him as a hypocrite. The truth is that ever

a
since his Bharat Jodo Yatra, Rahul Gandhi has done a very good job of projecting himself as a political leader

r
who is the exact opposite of Narendra Modi. He is love, compassion and humility to Modi’s autocracy, arrogance
and narcissism.

o
5O

p
tr-5S7T5U3O8O
1P

If anything spoiled Rahul’s show, it was his timing. He could not have chosen a worse moment to try and

T
convince the Indian diaspora in America that he is a worthy alternative to Narendra Modi. This is Modi’s moment
in the sun. The country that once denied him a visa to travel there is next week going to welcome him in a way
that few foreign leaders have been welcomed in the United States ever. There is not just a state banquet planned
but a private dinner with the Bidens in the White House, and when he addresses both houses of Congress for the
second time as prime minister, he will become one of a small handful of foreign leaders to be accorded this
honour. Some among the others are Winston Churchill, Nelson Mandela and Volodymyr Zelensky.

To add to Rahul’s timing problems came The Economist last week, with India on the cover as ‘America’s new
best friend’. The Economist is among the international newspapers that Modi’s media managers usually blame
for conspiring to defame India. The truth is that this caboodle of conspiracy theorists is more responsible for
giving Modi a bad name than any foreign publication. And the truth is that when Modi has deserved praise, he
has been given it in the very publications that are reviled for being ‘anti-India’ by BJP spokespersons.

94. Which among the following is the author most likely to agree with?
(a) Rahul Gandhi is a better leader
tr-5K7Ethan Narendra
5D3F8B 1E5J Modi for India’s future.
(b) Narendra Modi is the most popular and respected leader in the United States and the world.
(c) The Indian diaspora in America is influential and important for India’s interests and image.
(d) The BJP and its spokespersons are paranoid and biased against foreign media and criticism.

1P5O
tr-5S7T5U3O8O

Head Office: 127, Zone II, MP Nagar, Bhopal |+91-7676564400| https://www.toprankers.com Page 30 of 36
95. Which of the following is most similar to the author’s arguments in the given passage?
(a) A movie actor is praised for for originality, creativity and humour, but he is criticized for not working with
a famous director who directed a series of blockbuster movies.
(b) A sports analyst commends a young athlete for his talent, dedication and performance, and criticizes the
coach for not selecting him for a crucial match against a formidable opponent.
(c) A critic praises a movie for its novelty and audacity to showcase taboos, and explains why it failed to
perform at the box office.
(d) A food blogger compliments a new restaurant for its cuisine, service and ambience, but criticizes the owner
for opening it during the pandemic lockdown that limits its customers and revenue.

96. Which among the following option can be inferred from the passage?
(a) Rahul Gandhi is more popular than Narendra Modi among the Indian diaspora in America.
(b) Narendra Modi has been denied a visa to travel to the United States more than once.
(c) The Economist has praised Narendra Modi for his foreign policy achievements.
(d) Narendra Modi is the first Indian prime minister to address both houses of Congress.

c o m
97. Which among the following option captures the central idea of the passage?

rs .
(a) The passage compares and contrasts the personalities and political strategies of Rahul Gandhi and
Narendra Modi.

k e
(b) The passage criticizes Rahul Gandhi for his poor timing and lack of vision in challenging Narendra Modi.
(c) The passage praises Narendra Modi for his diplomatic success and popularity in the United States.

and Narendra Modi.

r an
(d) The passage analyzes the role of the media and the public opinion in shaping the image of Rahul Gandhi

98. Which of the following weakens the author’s


timing?

o
tr-5S 7T 5U3O8O1P5O

p
argument that Rahul Gandhi’s visit was spoiled because of the bad

(a) The contention that Narendra Modi is a more popular leader than Rahul Gandhi.

T
(b) The unwarranted assumption that Rahul’s impression on Indian diaspora was not bad.
(c) The premise that The Economist published India on the cover as ‘America’s new best friend’.
(d) The reasoning of the author’s argument comparing Rahul Gandhi with Narendra Modi.

Passage (Q.99-Q.103): The government of India is for the first time allowing universities to offer fully online
degrees -- a change that could reshape education delivery in the country while blowing open the door to a
previously limited market for U.S.-based online education services companies. Now, as part of a push to widen
access to higher education and raise the profile of Indian institutions globally, restrictions on online learning are
starting to lift. The government’s approach to online learning is, however, still cautious. Finance Minister
Nirmala Sitharaman spoke about the need to make India’s young people more employable through better higher
education opportunities. The government is currently working on a new national education policy. A draft
version of the policy outlines the important role online learning could play in reforming India’s education system
and expanding access to higher education. The policy encourages Indian institutions not only to develop their
own online programs, but also to recognize and award credit for online programs offered by foreign institutions.
The policy proposes that sometr-foreign 8B1E5J
5K7E5D3Finstitutions may be invited to operate in India -- something the country
has long resisted. In her speech, Sitharaman acknowledged that foreign investment in India’s education sector is
needed to “attract talented teachers, innovate and build better labs.” But few institutions have staff who are
experienced in launching online programs, and that has education service providers eyeing the subcontinent's
educational landscape eagerly.

Fully online 1P
degrees
5O can increase enrollment and completion, while at the same time reducing barriers to entry,
tr-5S7T5U3O8O
online degrees will likely be offered at lower cost than face-to-face programs and will likely appeal to working
adults who don’t have the time to pursue a traditional on-campus degree. An online Indian degree could also

Head Office: 127, Zone II, MP Nagar, Bhopal |+91-7676564400| https://www.toprankers.com Page 31 of 36
be attractive to students in South Asia, Africa or the Middle East. Currently most degrees in India are offered
by a single institution over two to four years.

99. Which of the following will strengthen government’s assertion that there is a pressing need for the change in
education system to make the youth more employable?
(a) By 2030, India is set to have the largest working-age population in the world.
(b) India’s youth not only need literacy, they need the medium to accessible and affordable
(c) India’s educated youth have been found to be severely deficient in both job and life skills
(d) Indian youth are naturally good in quantitative skills but are poor in language skills

100. Which of the following if true will NOT strengthen the idea in the statement “The government’s approach to
online learning is, however, still cautious”?
(a) Only the top 100 institutions in India’s National Institutional Ranking Framework can apply to offer fully
online degrees,

o m
(b) The subject areas in which the online degrees can be given are strictly restricted and grossly limited.

c
(c) Government has commented that there will be no online medical or law degrees from the country's
universities in the foreseeable future.

rs .
(d) For many years, Indian universities and colleges were not permitted to offer more than 20 percent of a
degree online

k e
101. Which of the following can construed to be reason for the government not allowing online degrees earlier?
(a) Concerns about quality of education

n
(b) Limited mechanisms for oversight and regulation of the system

a
(c) Poor computer infrastructure, limited bandwidth and internet connectivity
(d) All of the above

r
102. Which of the following willtr-most
to operate in India?
3O
5S7T5Ustrengthen

o p 8O1P5O
government’s contention that foreign universities need to be allowed

T
(a) Only 25 percent of students graduating from high school in India go on to pursue higher education which
the Indian government wants to increase to 50 percent
(b) Indian government wants to double the country’s college and university enrollment from its current base of
around 35 million students
(c) Few Indian colleges have the campus facilities or resources to accommodate a huge increase in students
over the next 15 years.
(d) With no financial support to build new facilities or open new universities, enrolling students online seems
the logical solution to boost capacity

103. Which of the following can inferred to be a benefit of complete online degree?
(a) Affordable and accessible
(b) Flexible for foreigners
(c) Company sponsored courses for working professionals
(d) Will improve the number of examinees
1E5J
tr-5K7E5D3F8B

1P5O
tr-5S7T5U3O8O

Head Office: 127, Zone II, MP Nagar, Bhopal |+91-7676564400| https://www.toprankers.com Page 32 of 36
Passage (Q.104-Q.108): In 1936, India’s first national park was named after Malcolm Hailey, the then governor
of the United Provinces. After Independence, it was renamed Ramganga, after the river that flows through the
park. In 1956, the protected area was rechristened once again, after Jim Corbett, the hunter-turned-naturalist,
whose name had, by then, become part of the forest’s lore — a rare instance of a public place being named for
an Englishman after Independence. Now Union Minister of State for Environment, Ashwini Kumar Choubey,
has proposed that the park do away with its association with the Englishman. Last week, Choubey reportedly
noted in the visitor’s book that he preferred the name “Ramganga National Park”. The Uttarakhand government
has reportedly opposed Choubey’s proposal, but the minister’s remarks have drawn justifiable outrage, and
stoked fears of a replay of the BJP government’s name-changing spree in the realm of conservation.

Names of public places, cities and streets, no doubt, have close links with the dominant political ideology of an
era. But acts of rechristening are most often driven by a simplistic and, at times chauvinistic, reading of the past,

m
one that is insensitive to the layered histories and identities of places. Born in Nainital to English parents, Corbett

o
volunteered for the British Army in both the World Wars. But his writings show him as steeped in the ecosystem

c
of Kumaon and Garhwal. They are suffused with empathy for both the people and nature. Writing about his

rs .
childhood friend, Kunwar Singh, Corbett recalls, for instance, that “We had a name for every outstanding tree,
for every water hole, game track, and nullah”. The naturalist also struck a friendship with freedom fighter and
UP’s first chief minister, Govind Ballabh Pant. It was on Pant’s insistence that the celebrated protected area

k e
came to be named after Corbett. In 1973, the park became the launchpad of Project Tiger — India’s first tiger
protection programme. In more ways than one, Corbett National Park is a testament to the well-known adage of
historians — the past lives in the present.

104. Author will disagree with:

r an
(a) Ashwini Kumar Choubey’s proposal to rename Corbett National Park strengthens the reserve’s rich history

tapestry tr-

o
5S 7T 5U3O8O1P5O

p
(b) Corbett National Park is not only a conservation reserve but a cultural heritage with a rich historical

(c) The settlements around Corbett National Park derive their livelihood from the tourists who visit the park

T
(d) The renaming of the Corbett National Park was done after Independence as homage to a person closely
associated with the area

105. Which of the following if true will strengthen the idea of the statement “In more ways than one, Corbett National
Park is a testament to the well-known adage of historians — the past lives in the present”?
(a) In and around the forests of Uttarakhand, guesthouses, general stores, gift shops, even saloons, carry
Corbett’s name.
(b) Uttarakhand’s forest minister Harak Singh Rawat has said that it would be “impractical to change the name
of the park”.
(c) He also added that Corbett “was a legend, a national pride” — a message for his colleague.
(d) All of the above
106. Which of the following if true will weaken author’s contention that the name of the Corbett Park shouldn’t be
changed?
(a) The name of many other forests reserves
tr-5K7E5D3F8B
1E5Jhas been changed at least three times since Independence.
(b) No other forest reserve in India is named after a foreigner
(c) Most other forest reserves in India are named after a river or a mountain in the region
(d) Recent evidence suggest that Corbett was involved in exploitation of the population in the area
107. Which of the following can definitely be inferred about Jim Corbett?
(a) He was the most famous hunter of his time who killed many rare animals
(b)
tr-5SHe 3O8Oone
7T5Uwas 1P5Oof the few British who participated in India’s freedom struggle

(c) He worked for the preservation of nature and wrote extensively on the subject
(d) He created a natural reserve in Uttaranchal for the preservation of the tigers
Head Office: 127, Zone II, MP Nagar, Bhopal |+91-7676564400| https://www.toprankers.com Page 33 of 36
108. Which of the following is an assumption by the author in the statement “the minister’s remarks have drawn
justifiable outrage, and stoked fears of a replay of the BJP government’s name-changing spree in the realm of
conservation”?
(a) People are entitled to interpret stray utterances by minister as indication of policy direction
(b) Ministers are prone to making wild and unsubstantiated allegations in the public forum to seek publicity
(c) Minister has the authority to change the name of a forest reserve as per his or her will without due process
(d) People respond viciously to any perceived attempt to alter the identity of a place by changing names

c o m
rs .
k e
r an
o p
tr-5S7T5U3O8O
1P5O

1E5J
tr-5K7E5D3F8B

1P5O
tr-5S7T5U3O8O

Head Office: 127, Zone II, MP Nagar, Bhopal |+91-7676564400| https://www.toprankers.com Page 34 of 36
SECTION - E : QUANTITATIVE TECHNIQUES

Direction (Q.109-Q.112): Ankur has some toys of shapes: sphere, cube, cylinder, cuboid and cone. The radius
of the sphere and cylinder are equal but the volume of the cylinder is 75% of the volume of the sphere. The
height of the cone is equal to the edge of the cube, and the volume of the cube is 21/22 times the volume of the
cone. The radius of the cone is equal to the height of the cuboid, and the height of the cuboid is half of the
diameter of the sphere. The ratio of volume of cuboid and cube is 132:529 respectively and the area of the base
22
of the cuboid is 132 cm². (Take   )
7
109. Find the radius of the cylinder in cm
(a) 23 (b) 25
(c) 27 (d) 28

110. Find the volume of the cube in cubic cm.


(a) 9261 (b) 12167

c o m
(c) 13824

111. Find the curved surface area (in sq cm) of the cone.
(d) None of these

rs .
(a) 448 √2𝜋
(c) 529 √2𝜋
(b) 441 √2𝜋
(d) 576 √2𝜋

k e
(a) 2 : π
(c) 1 : 2π
an
112. Find the ratio of the surface area of a cube and the total surface area of a cylinder.

r
(b) 3 : 2π
(d) 4 : 3π

o p
tr-5S7T5U3O8O
1P5O
Direction (Q.113-Q.117): Riya, a studious girl, was planning to go to her grandmother’s house during her

T
summer vacation. She decided to carry with her 5 books to study. She had two bundles of books, bundle 1
consists of five different books and bundle 2 consists of 4 different books. It is known that she picked at least 1
book from each bundle.

113. Find the number of ways in which she can carry books such that the number of books picked by her from
bundle 1 is more than that from bundle 2
(a) 75 (b) 80 (c) 85 (d) 90

114. Find the number of ways in which she can carry the books such that she picks at least 2 books from each
bundle.
(a) 90 (b) 95 (c) 100 (d) 105

115. Find the number of ways in which she can carry the books such that she picks at most 2 books from bundle
1 and at least 2 books from bundle 2
(a) 30 (b) 35 5D3F8B1E5J (c) 40 (d) 45
tr-5K7E

116. Find the number of ways in which she can carry the books such that the difference between the number of
books picked by her from both the bundles is 3
(a) 20 (b) 25 (c) 30 (d) 40

117. Find the number of ways in which she can carry books such that the number of books picked by her from
1P5O
tr-5S7T5U13Ois8Oless than that from bundle 2
bundle
(a) 45 (b) 55 (c) 40 (d) 54

Head Office: 127, Zone II, MP Nagar, Bhopal |+91-7676564400| https://www.toprankers.com Page 35 of 36
Direction (Q.118-Q.120): Following bar graph represents the sale of milk in 250 ml bottles and 500 ml bottles
from a store from Monday to Friday.
Total number of milk bottles sold = number of 250 ml, 500 ml and 750 ml milk bottles sold together
Sale of 750 ml milk bottles = 2 * (difference between sale of 250 ml and 500 ml milk bottles)

Sale of milk bottles


175

150

125

100

75

c o m
50

rs .
e
25

0
Monday

an
Tuesday

250 ml
Wednesday

k
500 ml
Thursday Friday

5S7T5U

p r
3O8O1P5O

o
118. What quantity (in liters) oftr-milk is sold from the store on Wednesday?
(a) 175 (b) 240 (c) 225 (d) 180

on Thursday?
(a) 5: 6
T
119. What is the ratio of sale of 500 ml milk bottles on Tuesday and Friday together to the sale of all milk bottles

(b) 3: 2 (c) None of these (d) 4: 5

120. If the average number of milk bottles sold on Wednesday and Saturday together is 438, then sale of milk
bottles on Saturday is increased or decreased by what percent as compared to that on previous day?
(a) 20% (b) 25% (c) 30% (d) 15%

1E5J
tr-5K7E5D3F8B

1P5O
tr-5S7T5U3O8O

Head Office: 127, Zone II, MP Nagar, Bhopal |+91-7676564400| https://www.toprankers.com Page 36 of 36

You might also like